You are on page 1of 38

Human Reproduction 795

CHAPTER 25

HUMAN REPRODUCTION

NCERT BIOLOGICS ©

REPRODUCTION IN HUMAN BEINGS inside by two types of cells called male germ cells
1. Human beings are sexually reproducing organisms and (spermatogonia) and Sertoli cells.
viviparous. 5. The male germ cells undergo meiotic divisions finally
2. The reproductive events in human beings include leading to sperm formation, while Sertoli cells provide
gametogenesis (formation of gametes) transfer of nutrition to the germ cells.
sperms into the female genital tract (insemination) and 6. The regions outside the seminiferous tubules called
fusion of male and female gametes (fertilisation) interstitial spaces, contain small blood vessels and
leading to zygote formation. interstitial cells or Leydig cells. Leydig cells
3. Zygote formation is followed by formation and synthesise and secrete testicular hormones called
development of blastocyst and its attachment to the androgens. Other immunologically competent cells are
uterine wall (implantation) embryonic development also present.
(gestation) and delivery of the baby (parturition). 7. The male sex accessory ducts include rete testis,
vasa efferentia, epididymis and vas deferens.
THE MALE REPRODUCTIVE SYSTEM The seminiferous tubules of the testis open into the
vasa efferentia through rete testis. The vasa efferentia
1. The male reproductive system is located in the pelvis leave the testis and open into epididymis located along
region. It includes a pair of testes alongwith the posterior surface of each testis. The epididymis
accessory ducts, glands and the external leads to vas deferens that ascends to the abdomen and
genitalia. loops over the urinary bladder. It receives a duct from
2. The testes are situated outside the abdominal cavity seminal vesicle and opens into urethra as the
within a pouch called scrotum. The scrotum helps in ejaculatory duct. These ducts store and transport the
maintaining the low temperature of the testes (2–2.5°C sperms from the testis to the outside through urethra.
lower than the normal internal body temperature) The urethra originates from the urinary bladder and
necessary for spermatogenesis. extends through the penis to its external opening called
3. In adults, each testis is oval in shape, with a length of urethral meatus.
about 4 to 5 cm and a width of about 2 to 3 cm. The 8. The penis is the male external genitalia . It is made up
testis is covered by a dense covering. Each testis has of special tissue that helps in erection of the penis to
about 250 compartments called testicular lobules. facilitate insemination. The enlarged end of penis
4. Each testicular lobule contains one to three highly called the glans penis is covered by a loose fold of skin
coiled seminiferous tubules in which sperms are called foreskin.
produced. Each seminiferous tubule is lined on its
796 NCERT Biology Booster

9. The male accessory glands include paired seminal tissue, which extend down from the mons pubis and
vesicles, a prostate and paired bulbourethral surround the vaginal opening. The labia minora are
glands. Secretions of these glands constitute the paired folds of tissue under the labia majora. The
seminal plasma which is rich in fructose, calcium and opening of the vagina is often covered partially by a
certain enzymes. The secretions of bulbourethral membrane called hymen. The clitoris is a tiny
glands also helps in the lubrication of the penis. finger-like structure which lies at the upper junction of
the two labia minora above the urethral opening. The
THE FEMALE REPRODUCTIVE SYSTEM hymen is often torn during the first coitus
(intercourse).
1. The female reproductive system consists of a pair of
ovaries alongwith a pair of oviducts, uterus, Note: Hymen can also be broken by a sudden fall or jolt,
cervix, vagina and the external genitalia located insertion of a vaginal tampon, active participation
in pelvic region. These parts of the system alongwith a in some sports like horseback riding, cycling, etc. In
pair of the mammary glands are integrated
some women the hymen persists even after coitus.
structurally and functionally to support the processes
In fact, the presence or absence of hymen is not a
of ovulation, fertilisation, pregnancy, birth and child
care. reliable indicator of virginity or sexual experience.
2. Ovaries are the primary female sex organs that produce 6. A functional mammary gland is characteristic of all
the female gamete (ovum) and several steroid female mammals. The mammary glands are paired
hormones (ovarian hormones). The ovaries are structures (breasts) that contain glandular tissue and
located one on each side of the lower abdomen. Each variable amount of fat. The glandular tissue of each
ovary is about 2 to 4 cm in length and is connected to breast is divided into 15-20 mammary lobes
the pelvic wall and uterus by ligaments. Each ovary containing clusters of cells called alveoli. The cells of
is covered by a thin epithelium which encloses the alveoli secrete milk, which is stored in the cavities
ovarian stroma. The stroma is divided into two zones (lumens) of alveoli. The alveoli open into mammary
a peripheral cortex and an inner medulla. tubules. The tubules of each lobe join to form a
3. The oviducts (fallopian tubes), uterus and vagina mammary duct. Several mammary ducts join to
constitute the female accessory ducts. Each fallopian form a wider mammary ampulla which is connected to
tube is about 10-12 cm long and extends from the lactiferous duct through which milk is sucked out.
periphery of each ovary to the uterus, the part closer to
the ovary is the funnel-shaped infundibulum. The GAMETOGENESIS
edges of the infundibulum possess finger-like It is the process of formation of sperm (male gamete) and
projections called fimbriae, which help in collection ovum (female gamete).
of the ovum after ovulation. The infundibulum leads to
a wider part of the oviduct called ampulla. The last Spermatogenesis
part of the oviduct, isthmus has a narrow lumen and 1. In testis, the immature male germ cells
it joins the uterus. (spermatogonia) produce sperms by
4. The uterus is single and it is also called womb. The spermatogenesis that begins at puberty.
shape of the uterus is like an inverted pear. It is 2. The spermatogonia (sing. spermatogonium)
supported by ligaments attached to the pelvic wall. The present on the inside wall of seminiferous tubules
uterus opens into vagina through a narrow cervix. The multiply by mitotic division and increase in numbers.
cavity of the cervix is called cervical canal which 3. Each spermatogonium is diploid and contains 46
alongwith vagina forms the birth canal. The wall of the chromosomes. Some of the spermatogonia called
uterus has three layers of tissue. The external thin primary spermatocytes periodically undergo
membranous perimetrium, middle thick layer of meiosis.
smooth muscle, myometrium and inner glandular
4. A primary spermatocyte completes the first meiotic
layer called endometrium that lines the uterine
division (reduction division) leading to formation of
cavity. The endometrium undergoes cyclical changes
two equal, haploid cells called secondary
during menstrual cycle while the myometrium exhibits
spermatocytes, which have only 23 chromosomes
strong contraction during delivery of the baby.
each. The secondary spermatocytes undergo the
5. The female external genitalia include mons pubis, labia second meiotic division to produce four equal, haploid
majora, labia minora, hymen and clitoris. Mons spermatids.
pubis is a cushion of fatty tissue covered by skin and
5. The spermatids are transformed into spermatozoa
pubic hair. The labia majora are fleshy folds of
(sperms) by the process called spermiogenesis.
Human Reproduction 797

After spermiogenesis, sperm heads become embedded Structure of Mature Sperm


in the Sertoli cells, and are finally released from the 1. It is a microscopic structure composed of a head,
seminiferous tubules by the process called neck, a middle piece and a tail. A plasma
spermiation. membrane envelops the whole body of sperm.
(46) Spermatogonia At Puberty 2. The sperm head contains an elongated haploid
Mitosis nucleus, the anterior portion of which is covered by a
differentiation cap-like structure, acrosome. The acrosome is filled
(46) Primary spermatocytes with enzymes that help fertilisation of the ovum.
1st meiotic 3. The middle piece possesses numerous mitochondria,
division which produce energy for the movement of tail that
(23) Secondary spermatocytes
facilitate sperm motility essential for fertilisation.
4. The human male ejaculates about 200 to 300 million
2nd meiotic
division sperms during a coitus of which, for normal
fertility, at least 60 per cent sperms must have
(23) Spermatids
normal shape and size and at least 40 per cent
Differentiation of them must show vigorous motility.
5. Sperms released from the seminiferous tubules, are
(23) Spermatozoa transported by the accessory ducts. Secretions of
epididymis, vas deferens, seminal vesicle and prostate
Fig. 25.1 Spermatogenesis
are essential for maturation and motility of sperms.
6. Spermatogenesis starts at the age of puberty due to The seminal plasma along with the sperms constitute
significant increase in the secretion of gonadotropin the semen. The functions of male sex accessory ducts
releasing hormone (GnRH)-hypothalamic hormone. and glands are maintained by the testicular hormones
The increased levels of GnRH then acts at the anterior (androgens).
pituitary gland and stimulates secretion of two Oogenesis
gonadotropins – luteinising hormone (LH) and follicle
1. The process of formation of a mature female gamete is
stimulating hormone (FSH).
called oogenesis.
7. LH acts at the Leydig cells and stimulates synthesis and
2. Oogenesis is initiated during the embryonic
secretion of androgens. Androgens, in turn, stimulate
development stage when a couple of million gamete
the process of spermatogenesis. FSH acts on the Sertoli
mother cells (oogonia) are formed within each
cells and stimulates secretion of some factors which
fetal ovary; no more oogonia are formed and
help in the process of spermiogenesis.
added after birth.
– 3. Oogonia start division and enter into prophase-I of the
Hypothalamus
meiotic division and get temporarily arrested at that
stage, called primary oocytes. Each primary oocyte
GnRH
then gets surrounded by a layer of granulosa cells and
– – is called the primary follicle. A large number of
Anterior pituitary
Negative feedback

these follicles degenerate during the phase from birth


to puberty. Therefore, at puberty only 60,000-80,000
Negative feedback

primary follicles are left in each ovary. The primary


FSH LH follicles get surrounded by more layers of granulosa
cells and a new theca and are called secondary
follicles.
Sertoli cells Leydig cells 4. The secondary follicle soon transforms into a tertiary
+ + + follicle which is characterised by a fluid filled cavity
Inhibin called antrum. The theca layer is organised into an
Spermato- Testosterone inner theca interna and an outer theca externa. It is at
genesis
this stage that the primary oocyte within the tertiary
T E S T I S follicle grows in size and completes its first meiotic
division. It is an unequal division resulting in the
Fig. 25.2 Hormonal control of the testes formation of a large haploid secondary oocyte and a
tiny first polar body . The secondary oocyte retains
798 NCERT Biology Booster

bulk of the nutrient rich cytoplasm of the primary + – –


Hypothalamus
oocyte.
5. The tertiary follicle further changes into the mature
follicle or Graafian follicle. The secondary oocyte GnRH
forms a new membrane called zona pellucida + – –
Anterior pituitary
surrounding it. The Graafian follicle now ruptures to
release the secondary oocyte (ovum) from the ovary by
the process called ovulation. FSH/LH

Oogonia (46) Ovary


Metosis
differentiation
Fetal life
Primary oocyte (46)
1st meiotic Estrogen
division Progesterone
Birth (completed prior Fig. 25.4 Hormonal control of female
childhood to ovulation) reproductive system
puberty Secondary
oocyte (23)
First MENSTRUAL CYCLE
Adult polar body
reproductive life Ovum (23) 1. The reproductive cycle in the female primates (e.g.,
monkeys, apes and human beings) is called menstrual
Second
polar body (23) cycle.
2. The first menstruation begins at puberty and is called
Fig. 25.3 Oogenesis menarche. In human females, menstruation is
repeated at an average interval of about 28/29 days,
Note: and the cycle of events starting from one menstruation
1. The advantage of unequal cytoplasmic till the next one is called the menstrual cycle. One
division in the primary oocyte : The unequal ovum is released (ovulation) during the middle of each
division in the primary oocyte results in the menstrual cycle.
formation of large haploid secondary oocyte and a
tiny polar body. The secondary oocyte retains the LH
FSH
bulk of the nutrient rich cytoplasm. In later stages
secondary oocyte divides to form the second tiny
polar body and a large ovum which retains most Developing
Mature Developing
follicle corpus luteum Repressing
cytoplasm required for early stages of development. follicle
corpus luteum
In humans, from one primary oocyte single ovum
and two polar bodies are formed. The ovum is
Estrogen
released from the ovary in the secondary oocyte
Progesterone
stage after the release of first polar body.
2. Does the first polar body born out of first
meiotic division divide further or
degenerate?
In most of the vertebrates including humans, the
first polar body does not undergo meiosis-II and
usually degenerate.
Fig. 25.5 Diagram showing various events
3. Usually, one ovum is released in each menstrual during a menstrual cycle
cycle (~28 days) by alternate ovaries, i. e., each
ovary releases six ova in a year. 3. The cycle starts with the menstrual phase, when
4. Sometimes, two or more follicles reach maturity in menstrual flow occurs and it lasts for 3-5 days.
one month cycle, so more than one oocyte may be 4. The menstrual flow results due to breakdown of
ovulated which can result in fraternal, i. e, non- endometrial lining of the uterus and its blood vessels
which forms liquid that comes out through vagina.
identical siblings.
Human Reproduction 799

Menstruation only occurs if the released into the uterus and finally reach the ampullary region
ovum is not fertilised. Lack of menstruation of the fallopian tube. The ovum released by the ovary is
may be indicative of pregnancy. However, it also transported to the ampullary region where
may also be caused due to some other fertilisation takes place.
underlying causes like stress, poor health etc. 2. The process of fusion of a sperm with an ovum is called
5. The menstrual phase is followed by the follicular phase. fertilisation. Fertilisation can only occur if the ovum
During this phase, the primary follicles in the ovary and sperms are transported simultaneously to the
grow to become a fully mature Graafian follicle and ampullary region. This is the reason why not all
simultaneously the endometrium of uterus regenerates copulations lead to fertilisation and pregnancy.
through proliferation. These changes in the ovary and 3. During fertilisation, a sperm comes in contact with the
the uterus are induced by changes in the levels of zona pellucida layer of the ovum and induces
pituitary and ovarian hormones. changes in the membrane that block the entry of
6. The secretion of gonadotropins (LH and FSH) increases additional sperms. Thus, it ensures that only one sperm
gradually during the follicular phase, and stimulates can fertilise an ovum. The secretions of the acrosome
follicular development as well as secretion of estrogens help the sperm enter into the cytoplasm of the ovum
by the growing follicles. Both LH and FSH attain a peak through the zona pellucida and the plasma membrane.
level in the middle of cycle (about 14th day). Rapid This induces the completion of the meiotic division of
secretion of LH leading to its maximum level during the the secondary oocyte. The second meiotic division is
mid-cycle called LH surge induces rupture of Graafian also unequal and results in the formation of a second
follicle and thereby the release of ovum (ovulation). polar body and a haploid ovum (ootid). Soon, the
7. The ovulation (ovulatory phase) is followed by the haploid nucleus of the sperms and that of the ovum
luteal phase during which the remaining parts of the fuse together to form a diploid zygote.
Graafian follicle transform as the corpus luteum.
The corpus luteum secretes large amounts of Note: All the haploid gametes produced by the female
progesterone which is essential for maintenance of the (ova) have the sex chromosome X whereas, in the
endometrium. Such an endometrium is necessary for male gametes (sperms) the sex chromosome could
implantation of the fertilised ovum and other events of be either X or Y, hence, 50 per cent of sperms carry
pregnancy. During pregnancy all events of the the X chromosome while the other 50 per cent carry
menstrual cycle stop and there is no menstruation. In the Y. After fusion of the male and female gametes
the absence of fertilisation, the corpus luteum the zygote would carry either XX or XY depending
degenerates. This causes disintegration of the on whether the sperm carrying X or Y fertilised the
endometrium leading to menstruation, marking a new ovum. The zygote carrying XX would develop into a
cycle.
female baby and XY would form a male . So
8. In human beings, menstrual cycles ceases around 50
scientifically the sex of baby is determined by father
years of age; that is termed as menopause. Cyclic
not by the mother.
menstruation is an indicator of normal reproductive
phase and extends between menarche and menopause. 4. The mitotic division starts as the zygote moves through
the isthmus of the oviduct called cleavage towards
Note: Menstrual Hygiene : Maintenance of hygiene the uterus and forms 2, 4, 8, 16 daughter cells called
and sanitation during menstruation is very blastomeres. The embryo with 8 to 16 blastomeres is
important. One should take bath and clean called a morula.
regularly. Sanitary napkins or clean home made 5. The morula continues to divide and transforms into
pads should be used and must be changed after blastocyst as it moves further into the uterus. The
every 4-5 hours as per requirement. Napkins should blastomeres in the blastocyst are arranged into an
be disposed by properly wrapping it with a used outer layer called trophoblast and an inner group of
paper. These should not be thrown in the drain pipe cells attached to trophoblast called the inner cell
of toilets or open area. After handling the napkin mass. The trophoblast layer then gets attached to the
hands must be washed with soap. endometrium and the inner cell mass gets
differentiated as the embryo. After attachment, the
uterine cells divide rapidly and covers the blastocyst. As
FERTILISATION AND IMPLANTATION
a result, the blastocyst becomes embedded in the
1. During copulation (coitus) semen is released by the endometrium of the uterus. This is called
penis into the vagina (insemination). The motile implantation and it leads to pregnancy.
sperms swim rapidly, pass through the cervix, enter
800 NCERT Biology Booster

PREGNANCY AND EMBRYONIC are formed, for example, the limbs and
DEVELOPMENT external genital organs are well developed.
8. The first movements of the foetus and appearance of
1. After implantation, finger-like projections appear on hair on the head are usually observed during the fifth
the trophoblast called chorionic villi which are month. By the end of about 24 weeks (end of second
surrounded by the uterine tissue and maternal blood. trimester), the body is covered with fine hair, eye-lids
2. The chorionic villi and uterine tissue become separate, and eyelashes are formed. By the end of nine
interdigitated with each other and jointly form a months of pregnancy, the foetus is fully developed and
structural and functional unit between developing is ready for delivery.
embryo (foetus) and maternal body called placenta.
3. The placenta facilitate the supply of oxygen and PARTURITION AND LACTATION
nutrients to the embryo and also removal of carbon
dioxide and excretory/waste materials produced by the Parturiton
embryo. 1. The average duration of human pregnancy is about 9
4. The placenta is connected to the embryo through an months which is called the gestation period.
umbilical cord which helps in the transport of 2. Vigorous contraction of the uterus at the end of
substances to and from the embryo. Placenta also acts pregnancy causes expulsion/delivery of the foetus. This
as an endocrine tissue and produces several hormones process of delivery of the foetus (childbirth) is called
like human chorionic gonadotropin (hCG), parturition.
human placental lactogen (hPL), estrogens, 3. Parturition is induced by a complex neuroendocrine
progestogens, etc. In the later phase of pregnancy, a mechanism. The signals for parturition originate from
hormone called relaxin is also secreted by the ovary. the fully developed foetus and the placenta which
5. hCG, hPL and relaxin are produced in women only induce mild uterine contractions called foetal
during pregnancy. In addition, during pregnancy the ejection reflex. This triggers release of oxytocin from
levels of other hormones like estrogens, progestogens, the maternal pituitary. Oxytocin acts on the uterine
cortisol, prolactin, thyroxine, etc., are increased muscle and causes stronger uterine contractions, which
severalfolds in the maternal blood. Increased in turn stimulates further secretion of oxytocin. The
production of these hormones is essential for stimulatory reflex between the uterine contraction and
supporting the fetal growth, metabolic changes in the oxytocin secretion continues resulting in stronger and
mother and maintenance of pregnancy. stronger contractions. This leads to expulsion of the
6. Immediately after implantation, the inner cell mass baby out of the uterus through the birth canal i. e.,
(embryo) differentiates into an outer layer called parturition. Soon after the infant is delivered, the
ectoderm and an inner layer called endoderm. A placenta is also expelled out of the uterus.
mesoderm soon appears between the ectoderm and
Lactation
the endoderm. These three layers give rise to all tissues
(organs) in adults. It needs to be mentioned here that 1. The mammary glands of the female undergo
the inner cell mass contains certain cells called stem differentiation during pregnancy and starts producing
cells which have the potency to give rise to all the milk towards the end of pregnancy by the process
tissues and organs. called lactation. This helps the mother in feeding the
7. The human pregnancy lasts for 9 months. In human newborn.
beings, after one month of pregnancy, the 2. The milk produced during the initial few days of
embryo’s heart is formed. The first sign of lactation is called colostrum which contains several
growing foetus may be noticed by listening to the heart antibodies absolutely essential to develop resistance
sound carefully through the stethoscope. By the end of for the new-born babies. Breast-feeding during the
the second month of pregnancy, the foetus develops initial period of infant growth is recommended by
limbs and digits. By the end of 12 weeks (first doctors for bringing up a healthy baby.
trimester), most of the major organ systems
Human Reproduction 801

Section-A : Questions Based on NCERT Textbook


The Male Reproductive System Ureter Urinary
bladder
1. Identify A, B, C, D, E and F in the sectional view of male
Vas deferens
pelvis showing the reproductive system : D
E
F
A A
B Urethra
B C
Urinary bladder Testis
Testicular
C Ejaculatory lobules
Prostate duct
Glans
Penis penis Foreskin
D Rectum

Glans Penis Anus Opti


A B C D E F
E ons
Epidid- Rete testis Vasa Seminal Prostate Bulbo-
Scrotum Testis (a)
F ymis efferentia vesicle gland urethral
gland
Opti Vasa- Epididymis Rete Seminal Prostrate Bulbo-
A B C D E F (b) efferentia testes vesicle gland urethral
ons gland
Urethra Seminal Vas Ureter Foreskin Bulbo- Rete Vasa Epidi- Seminal Prostate Bulbo-
(a) vesicle deferens urethral (c) testis efferentia dymis vesicle gland urethral
gland gland
Ureter Seminal Vas Urethra Prepuce Bulbo- Epidi- Vasa Rete Seminal Prostate Bulbo-
(b) vesicle deferens urethral (d) dymis efferentia testis vesicle gland urethral
gland gland
Ureter Bulbo- Vas Urethra Prepuce Seminal
(c) urethral gland deferens vesicle 5. Read the following statements :
Urethra Bulbo- Vas Ureter Foreskin Seminal
(d) urethral gland deferens vesicle
(i) Each testis has about 25 compartments called
testicular lobules
2. Choose the correct statement : (ii) Each testicular lobule contains one to three highly
(a) Size of testis is 4-5 cm in length and 1 cm in width coiled seminiferous tubules in which sperms are
(b) The scrotum is maintained at body temperature produced
(c) The testes are situated outside the abdominal (iii) Sertoli cells acts as ‘nurse cells’ of testicles
cavity in humans (iv) Sertoli cells are activated by FSH secreted by the
(d) The earliest stages of spermatogenesis occur adenohypophysis
closest to the lumen of seminiferous tubules (v) Each testis is oval shaped with a length of about
3. Which of the following is not properly paired ? 2-3 cm and width of about 1-3 cm in adults
(a) Sertoli cells — Follicle cells How many of the above statements are correct ?
(b) Vas deferens — Oviduct (a) Three (b) Two
(c) Scrotum — Labia majora (c) Four (d) Five
(d) Seminiferous — Cervix 6. In males, excretory and reproductive systems share
tubule the:
4. Identify the parts A, B, C, D, E and F in the (a) Testes (b) Vas deferens
diagrammatic view of male reproductive system : (c) Seminal vesicle (d) Urethra
802 NCERT Biology Booster

7. Seminiferous tubule : 15. Which of the following is correct for A, B, C and D :


(a) Is lined by two types of cells called spermatogonia Ureter Urinary
and sertoli cells bladder
(b) Contains huge number of Leydig cells inside it A
C
(c) Contains huge number of interstitital cells inside it
Prostate
(d) Open directly into vasa efferentia D
8. Leydig cells :
B
(a) Are also called interstitial cells Vasa efferentia Urethra
(b) Synthesise and secrete androgens Rete testis
(c) Are immunologically competent cells
Testicular Testis
(d) Are related to more than one option lobules
9. The regions just outside the seminiferous tubules contain ? Glans
(a) Small blood vessels but lack interstitial cells penis Foreskin

(b) Cells which secrete testosterone


Opti
(c) Cells which get stimulated by LH of pituitary gland ons
A B C D
(d) Both (b) and (c)
(a) Carries Stores sperm Secretes a Secretes a
10. Choose the odd one out w.r.t. male sex accessory ducts: sperm permanently fluid that fluid that
(a) Vas deferens lubricates ultimately
(b) Vasa efferentia and epididymis ureter becomes
semen
(c) Rete testis
(b) Carries Stores sperm Carries urine Stores sperm
(d) Seminiferous tubules sertoli cells
11. The seminiferous tubules open into ___________
(c) Carries Stores sperm Secretes a Secretes a
through ____________ . sperm fluid that fluid that
(a) Vasa efferentia; epididymis ultimately lubricates
(b) Testicular tubules; epididymis becomes distal
semen urethra
(c) Vasa efferentia; rete testes
(d) Carries Stores sperm Secretes a Secretes a
(d) Rete testes; vasa efferentia sperm permanently fluid that fluid that
12. The vasa efferentia leave the testes and open into : lubricates ultimately
(a) Epididymis located along the posterior surface of distal urethra becomes
semen
each testis
(b) Epididymis located along the anterior surface of 16. Cowper’s gland is :
each testis (a) Located below prostate gland
(c) Rete testis (b) Also known as bulbourethral gland
(d) Testicular lobules (c) Located in between seminal vesicle and prostate
13. Which of the following is true ? gland
(a) Vasectomy inhibits spermatogenesis (d) Both (a) and (b)
(b) Vasectomy prevents sperm from exiting the male 17. Arrange the following structures in sequence through
urethra which the sperm moves :
(c) Blockage of vasa efferentia prevents sperm (i) Vasa efferentia
transport from testis to epididymis (ii) Epididymis
(d) Both (b) and (c) (iii) Urethra
14. Vas deferens : (iv) Seminiferous tubules
(a) Receives a duct from the seminal vesicle and opens (v) Vas deferens
into urethra as ejaculatory duct (vi) Rete testis
(b) Store sperms but do not transport them (vii) Ejaculatory duct
(c) Transport the fluid lacking sperms (a) (iv) ® (vi) ® (ii) ® (i) ® (vii) ® (v) ® (iii)
(d) Originates from the urinary bladder (b) (iv) ® (vi) ® (i) ® (v) ® (ii) ® (iii) ® (vii)
Human Reproduction 803

(c) (iv) ® (i) ® (vii) ® (iii) ® (ii) ® (vi) ® (v) 21. Find out which of the following statements are true
(d) (iv) ® (vi) ® (i) ® (ii) ® (v) ® (vii) ® (iii) (T)/ false (F) and choose the correct option :
18. The male urethra : I. The head of epididymis is called caput epididymis
(a) Originates from the urinary bladder II. The scrotum helps in maintaining the low
(b) Extends through the penis to external opening temperature of testes (2–2.5°C) lower than the
called meatus internal body temperature) for spermatogenesis
(c) Both (a) and (b) III. Prostate secretion inactivates sperm
(d) Can eject urine but not semen IV. In most mammals testes is located in the scrotal sac
19. Read the following statements : Opti
(i) The seminal plasma is rich in fructose and I II III IV
ons
calcium but lack enzymes T T F T
(a)
(ii) Penis is made of a special erectile tissue which
(b) T T T T
facilitates insemination
(iii) The enlarged end of penis is called glans penis (c) T F T F
(iv) The secretion of bulbourethral glands helps in the (d) T F F F
lubrication of the penis
(v) The male accessory sex-glands include seminal 22. Leydig cells :
vesicles, prostate, paired bulbourethral glands (a) Produce testosterone in the presence of luteinizing
and testis hormone
How many of the above statements are incorrect ? (b) Nourish spermatozoa
(a) Two (b) Three (c) Are present inside the seminiferous tubules
(c) Four (d) One (d) Secrete Androgen Binding Protein (ABP)
20. Identify A, B, C and D in the diagrammatic sectional 23. Choose the correct statement :
view of seminiferous tubule : (a) Epididymis is a long coiled tube present between
vasa efferentia and vas deferens
(b) Scrotal sacs are connected with the abdominal
cavity by inguinal canal
(c) Cryptorchidism is failure of one or both testicles to
A descend into the scrotum
(d) More than one option is correct
B 24. In which of the following structures sperms undergo
maturation ?
(a) Epididymis
C (b) Seminiferous tubules
(c) Vasa efferentia
D
(d) Rete testes
Opti The Female Reproductive System
A B C D
ons
25. Which of the following is incorrect ?
(a) Sertoli cells Spermato- Spermatozoa Interstitial
(a) Each ovary is about 2 to 4 cm in length and is
gonia cells
connected to the pelvic wall and uterus by
(b) Interstitial Spermato- Spermatozoa Sertoli cells
cells gonia
ligaments
(b) Each ovary is covered by a thin epithelium which
(c) Spermato- Interstitial Spermatozoa Sertoli cells
gonia cells encloses the ovarian stroma
(d) Spermato- Spermato- Interstitial Sertoli cells (c) Ovaries produce several steroid hormones but not
zoa gonia cells a single protein hormone
(d) The stroma of ovary is divided into a peripheral
cortex and an inner medulla
804 NCERT Biology Booster

26. Choose the incorrect match : 31. In the fallopian tube, the edges of infundibulum
(a) Oestrogen — Steroid possess finger-like projections called :
(b) Relaxin — Steroid (a) Fimbrae (b) Isthmus
(c) Progesterone — Steroid (c) Ampulla (d) Primordium
(d) Inhibin — Protein 32. Identify A, B, C, D, E, F and G in the diagrammatic
27. Identify A, B, C, D, E and F in the diagrammatic sectional view of the female reproductive system :
sectional view of female pelvis showing reproductive Uterine cavity
system : D

E
A F
E
A
Urinary Rectum B Ovary
Fimbriae
bladder C G
Pubic
Cervical canal
symphysis F
Urethra
B Anus Opti
A B C D E F G
C ons
(a) Endo- Myo- Peri- Isthmus Ampulla Infundi- Vagina
D Vaginal orifice metrium metrium metrium bulum
(b) Endo- Peri- Myo- Isthmus Ampulla Infundi- Cervix
metrium metrium metrium bulum
Opti
A B C D E F (c) Peri- Myo- Endo- Isthmus Ampulla Infundi- Cervix
ons
metrium metrium metrium bulum
Uterus Clitoris Labium Labium Cervix Vagina Endo- Myo- Peri- Isthmus Ampulla Infundi- Cervix
(a) minora majora
(d)
metrium meterium metrium bulum
Uterus Clitoris Labium Labium Cervix Vagina
(b) majora minora 33. Which of the following help in collection of the ovum
Uterus Clitoris Labium Labium Vagina Cervix
(c) minora majora
immediately after ovulation ?
Uterus Vagina Labium Labium Clitoris Cervix (a) Fimbriae (b) Ampulla
(d) minora majora
(c) Isthmus (d) Cervix
28. The vulva : 34. Match the columns :
(a) Consists of the external genital organs of a woman Column-I Column-II
(b) Includes mons pubis, labia majora, labia minora, A. Oviduct (i) Narrow lumen
clitoris and hymen
B. Isthmus (ii) Womb
(c) Both (a) and (b)
C. Uterus (iii) Fallopian tube
(d) Includes mons pubis, labia majora, labia minora,
D. Infundibulum (iv) Funnel shaped
clitoris and vagina
29. Mesovarium : (a) A = (i), B = (iv), C = (iii), D = (ii)
(a) Is the portion of broad ligament of uterus that (b) A = (ii), B = (i), C = (iii), D = (iv)
suspends the ovaries (c) A = (iii), B = (i), C = (ii), D = (iv)
(b) Fully covers the ovaries (d) A = (iv), B = (ii), C = (i), D = (iii)
(c) Both (a) and (b) 35. The part of fallopian tube near uterus is called
(d) Is ectodermal in origin _________ and near ovary is called ________ .
30. The lower narrow end of uterus is called : (a) Uterus; Isthmus
(a) Cervix (b) Cervical canal; Isthmus
(b) Fimbrae (c) Isthmus; Ampulla
(c) Urethra (d) Isthmus; Infundibulum
(d) Fallopian tube
Human Reproduction 805

36. The female external genetalia do not include : (b) A cushion of fatty tissue covered by skin and pubic
(a) Labia majora, labia minora, mons pubis and hymen hair
(b) Hymen, clitoris and mons
(c) Labia majora and labia minora only (c) The pubic bone in females
(d) Vagina and cervix (d) The internal genitalia of females
37. Read the following statements : 43. Identify the labelled parts A, B, C, D and E in the
(i) The uterus is present in single pair and is also diagrammatic sectional view of mammary gland :
called womb Fat
(ii) The shape of uterus is like an inverted pear Mammary lobe
(iii) The uterus opens into vagina through a narrow A
B Rib
cervix
C
(iv) Birth canal is formed by cervical canal and vagina
(v) Perimetrium of uterus wall is external and thick, D
Muscles
myometrium is middle thin layer of smooth between ribs
muscles and endometrium is inner glandular
layer Nipple Pectoralis
Which of the above statements are correct ? major muscle
(a) (i), (ii), (iii) (b) (ii), (iii), (iv) E
(c) (iv), (v) (d) (i), (v)
38. The endometrium of uterus :
(a) Undergoes cyclical changes during menstrual cycle
Opti
(b) Has a functional layer that thickens and then is A B C D E
ons
sloughed during menstrual cycle
(a) Ampulla Lactiferous Alveolus Areola Mammary
(c) Exhibits strong contraction during delivery of the duct duct
baby (b) Alveolus Mammary Ampulla Lactiferous Areola
(d) More than one option is correct duct duct

39. The hymen : (c) Areola Mammary Ampulla Lactiferous Alveolus


duct duct
(a) Gets torn only as a result of first coitus
(d) Lactiferous Mammary Ampulla Areola Alveolus
(b) Gets broken by sudden fall or jolt or by insertion of duct duct
vaginal tampon
(c) Persists in all women even after coitus 44. Find out the correct pathway of milk flow in the breast :
(d) In ruptured form is always a reliable indicator of I. Mammary duct
loss of virginity II. Lactiferous duct
40. The lower narrow end of womb is called : III. Mammary tubules
(a) Vulva IV. Alveoli lumen
(b) Ureter V. Mammary ampula
(c) Clitoris (a) IV ® III ® V ® I ® II
(d) Cervix (b) III ® I ® IV ® II ® V
41. The clitoris is : (c) IV ® III ® I ® V ® II
(a) Represented by hymen (d) IV ® III ® II ® I ® V
(b) A male secondary sex organ 45. The glandular tissue of each breast is divided into
(c) A tiny finger-like structure that lies at the upper ___________ containing clusters of cells called alveoli :
junction of the two labia minora (a) 2-3 mammary lobes
(d) A tiny finger-like structure present below the (b) 15-20 mammary ducts
urethral opening in females (c) 2-3 mammary ducts
42. Mons pubis is : (d) 15-20 mammary lobes
(a) Exclusively a male external genetalia
806 NCERT Biology Booster

Gametogenesis (c) Two equal Spermatids Four equal Secondary


spermatocytes
46. Identify A, B, C and D from the diagrammatic sectional
view of seminiferous tubule : (d) Two equal Spermatids Four Secondary
unequal spermatocytes
A
50. The transformation of spermatids into spermatozoa
(sperms) is called :
B (a) Spermatogenesis
(b) Spermiogenesis
(c) Spermiation
C (d) All of the above
D
51. Choose the correct statement :
(a) After spermiogenesis sperm heads become
Spermatogonium embedded in the sertoli cells
(b) The release of sperms from the seminiferous
Opti tubules is called spermiation
A B C D
ons
(c) In testis, the immature male germ cells
(a) Spermato Mature Primary Sertoli cell (spermatogonia) produce sperms by
- zoa spermatid spermatocyte spermatogenesis that begin at puberty
(b) Sertoli Spermatozoa Secondary Primary (d) More than one option is correct
cell oocyte spermatocyte
52. The acrosome of sperm :
(c) Spermato Secondary Mature Sertoli cell
- zoa spermatocyte spermatid
(a) Is filled with enzymes that help fertilization
(b) Is derived from Golgi apparatus
(d) Spermato Secondary Primary Sertoli cells
- zoa spermatocyte spermatocyte (c) Both (a) and (b)
(d) Is a modified endoplasmic reticulum
47. Spermatogonia : 53. Identify A, B, C, D and E in the diagrammatic structure
(a) Are present outside the seminiferous tubules of sperm :
(b) Increase in number by mitosis
(c) Are haploid, each with 46 chromosomes B
(d) Do not undergo mitosis
A C
48. Primary spermatocyte :
(a) Is diploid
(b) Undergo meiosis to produce four equal sized D
haploid spermatids
(c) Both (a) and (b)
(d) Possess 23 chromosomes
E
49. A primary spermatocyte completes meiosis-I resulting
(A) (B)
in the formation of ¾¾¾ haploid cells called ¾¾¾ .
The secondary spermatocytes undergo meiosis-II to
(C) (D)
produce ¾¾¾ haploid cells called ¾¾¾ .

Opti
A B C D
ons

(a) Two Secondary Four equal Spermatids


unequal spermatocytes
(b) Two equal Secondary Four equal Spermatids
spermatocytes
Human Reproduction 807

Opti 58. Choose the incorrect statement :


A B C D E
ons (a) Oogonia are formed and added after birth
(a) Head Nucleus Acrosome Neck Mitochondria (b) Oogenesis is initiated during the embryonic
development
(b) Head Acrosome Nucleus Neck Mitochondria
(c) No more gamete mother cells are formed in
(c) Tail Acrosome Nucleus Neck Mitochondria females after birth
Tail Acrosome Nucleus Middle Mitochondria (d) All of the above
(d)
piece 59. The tertiary follicle (in the ovary) is characterised by a
fluid filled cavity called :
54. Which part of a sperm possess numerous
(a) Atrium (b) Ostiole
mitochondria?
(c) Orifice (d) Antrum
(a) Neck (b) Tail
60. Read the following statements w.r.t. oogenesis :
(c) Middle piece (d) Head
(i) Oogenesis is remarkably same as
55. Find out which of the following statements are true
spermatogenesis
(T)/ false (F) and choose the correct option :
(ii) The oogonial cells starts division and enter into
I. Human male ejaculates about 200 to 300 million
prophase-I of meiosis and get temporarily
sperms during a coitus
arrested, called primary oocyte
II. For normal fertility, atleast 60% sperms must
(iii) Each primary oocyte gets surrounded by a layer of
have normal shape and size and atleast 40% of
granulosa cells and called primary follicle
them must show vigorous motility
(iv) The primary follicle get surrounded by layers of
III. Sperms released from the seminiferous tubules,
granulosa cells and new theca and called
are transported by the accessory ducts
secondary follicle
IV. The seminal plasma alongwith the sperms
(v) A large number of primary follicles degenerate
constitute the semen
from birth to puberty and at puberty only 600 to
Opti 800 primary follicles are left in each ovary
I II III IV
ons How many of the above statements are incorrect ?
(a) T T T F (a) Three (b) Two
(c) One (d) Four
(b) T T T T
61. At the onset of puberty :
(c) F F T T (a) Primary follicles begin to mature within each
T F T F ovarian cycle to form secondary follicle and then
(d)
tertiary follicle
56. The maturation and motility of sperms is governed by (b) Primary oocyte within the tertiary follicle grows in
the secretions of : size and completes its meiotic division
(a) Epididymis (c) Both (a) and (b)
(b) Vas deferens (d) Primary ooctyes develop from oogonial cells
(c) Seminal vesicle and prostate 62. The first meiotic division of primary oocyte in the
(d) Prostate, seminal vesicle, epididymis and vas tertiary follicle :
deferens (a) Is an equal division resulting in the formation of
57. The _________ of sperm possess numerous _________, secondary oocyte and first polar body
which provides energy for the movement of tail that (b) Results in formation of a secondary oocyte and a
facilitates sperm motility for fertilization : polar body where the secondary oocyte retains
(a) Head ; mitochondria bulk of nutrient rich cytoplasm
(b) Neck ; mitochondria (c) Both (a) and (b)
(c) Middle piece; mitochondria (d) Directly results in ova formation
(d) Middle piece; endoplasmic reticulum
808 NCERT Biology Booster

63. Identify the structures labelled as A, B, C, D and E in the 68. Choose the correct statement :
diagrammatic section of an ovary : (a) Corpus luteum degenerates to form corpus
B albicans if egg does not get fertilized
A C (b) In the absence of pregnancy corpus luteum
degenerates
D
(c) The tertiary follicle matures into Graafian follicle
(d) More than one option is correct
69. Read the following statements :
(i) Spermatogonia and spermatids are diploid
Ovum (ii) The extrusion of second polar body from egg
nucleus occurs after the entry of sperm and before
E completion of fertilization
Opti (iii) Spermatogenesis and sperm differentiation are
A B C D E under the control of FSH and testosterone
ons
Blood Primary Tertiary Graafian Corpus luteum (iv) A change in ovum after penetration of sperm is
(a)
vessels follicle follicle follicle the formation of second polar body
with
antrum
(v) The secondary oocyte in the Graafian follicle
Corpus Primary Tertiary Graafian Blood vessels
forms a new membrane called zona pellucida
(b)
luteum follicle follicle follicle surrounding it
(c) Blood Primary Tertiary Graafian Corpus luteum Which of the above statements are correct ?
vessels follicle follicle follicle (a) (i), (ii), (iii), (iv) (b) (i), (iii)
without
antrum (c) (iii) only (d) (ii), (iii), (iv), (v)
(d) Blood Primary Tertiary Graffian Corpus luteum 70. Identify the stages labelled as A, B, C and D in the
vessels follicle follicle follicle schematic diagram of spermatogenesis :
64. In human beings : Spermatogonia At Puberty
(a) The first polar body do not undergo meiosis-II and A
usually degenerates
(b) From one primary oocyte, a single ovum and three Primary spermatocytes
polar bodies are formed B
(c) From one primary oocyte, two ova and two polar
Secondary spermatocytes
bodies are formed
(d) The first polar body undergoes meiosis-I C
65. How many ovum/ova are released during the
Spermatids
formation of fraternal and maternal twins respectively?
(a) Two and one (b) One and two D
(c) Two and two (d)One and one Spermatozoa
66. The hormone required for ovulation and development
of corpus luteum is : Opti
A B C D
(a) TSH (b) FSH ons
(c) LH (d) PRL (a) Mitosis and Meiosis-I Meiosis-II Differentiation
67. The egg after coming out of ovary enters the fallopian differentiation
tube in : (b) Mitosis Meiosis-I Meiosis-II Mitosis
(a) Mature stage Meiosis-I Meiosis-I Mitosis Differentiation
(c)
(b) Oogonial cell stage
(d) Mitosis and Mitosis Meiosis-I Meiosis-II
(c) Secondary oocyte stage
differentiation
(d) Primary oocyte stage
Human Reproduction 809

71. How many of the following structures in the box 75. Identify A, B, C, D and E in the table provided :
contain 46 chromosomes ?
Cell type Ploidy DNA content
Spermatozoa, secondary oocyte, spermatogonia, secondary
spermatocyte, primary spermatocyte, spermatid, polar Oogonium A 2C or 4C
body, primary oocyte, oogonia Primary oocyte 2n B

(a) Six (b) Four Secondary oocyte C D


(c) Nine (d) Seven Ovum n E
72. In humans, during ovulation :
Opti
(a) The Graafian follicle ruptures to release the A B C D E
ons
primary oocyte
(a) 2n 2C n 2C 1C
(b) The Graafian follicle do not rupture
(c) Sharp decline in the level of LH and FSH occurs (b) 4n 4C n 2C 1C

(d) The Graafian follicle ruptures to release the n 2C n 2C 1C


(c)
secondary oocyte
(d) 2n 4C n 2C 1C
73. What is the advantage of unequal cell division
(meiosis) during the formation ovum ?
(a) Mature ovum retains most of the cytoplasm 76. Which of the following parts of a human sperm
possesses the typical ‘9 + 2’ microtubular organisation ?
(b) Retention of more cytoplasm in ovum provides it
with ample nutrients during development (a) Middle piece (b) Tail
(c) Retention of more cytoplasm in ovum allow it to (c) Both (a) and (b) (d) Head
survive for long duration 77. The ovulation in human females is :
(d) More than one option is correct (a) Followed by luteal phase
74. Identify A, B, C and D in the schematic representation (b) Followed by the phase during which Graafian
of oogenesis : follicle transforms into corpus luteum
Oogonia (c) Both (a) and (b)
(d) Followed by the formation of corpus luteum which
A
Fetal life secretes negligible amounts of progesterone
Primary oocyte 78. Which of the following is released from an ovary ?
(a) Secondary oocyte
B
Birth (b) Graafian follicle
childhood
puberty Secondary
(c) Primary oocyte
oocyte (d) Oogonial cells
C
79. Find out the correct sequence of spermatogenesis :
Adult
reproductive life Ovum I. Spermatid
D II. Spermatogonia
III. Secondary spermatocyte
Opti IV. Primary spermatocyte
A B C D
ons V. Sperms
(a) Mitosis and Meiosis-I and First polar Second polar (a) II ® IV ® III ® I ® V
differentiation meiosis-II body body (b) II ® III ® IV ® I ® V
(b) Mitosis Mitosis Ovum Ovum (c) II ® I ® V ® III ® IV
Mitosis and Meiosis-I First polar Second polar (d) II ® IV ® III ® V ® I
(c)
differentiation completed body body 80. The number of sperm(s) formed from a secondary
prior to (A)
spermatocyte is ¾¾¾ whereas, the number of ovum/
ovulation
(B)
(d) Meiosis-I Meiosis-I First polar Second polar ova formed from a secondary oocyte is ¾¾¾ .
completed body body (a) A = 2, B = 1 (b) A = 3, B = 1
prior to
(c) A = 1, B = 2 (d) A = 4, B = 1
ovulation
810 NCERT Biology Booster

81. For which of the following is the number the same in 85. Menstrual flow :
spermatogenesis and oogenesis ? (a) Occurs only if the released ovum gets fertilized
(a) Functional gametes produced by meiosis (b) If does not occur, is always an indication of
(b) Meiotic divisions required to produce gametes pregnancy
(c) Different cell types produced by meiosis (c) Results due to the breakdown of endometrial lining
(d) Gametes produced in a given time period of the ureter
(d) Sheds the lining of womb which passes out through
Menstrual Cycle
vagina
82. The first menstruation begins at puberty and is called :
86. Lack of menstruation :
(a) Menarche
(a) May be an indicative of pregnancy
(b) Menopause
(b) May be caused due to stress and poor health
(c) Menses
(c) Either (a) or (b)
(d) Menstruation cycle
(d) Is indicative of menopause in young women
83. In human females :
87. Find out which of the following statements are true
(a) Menstruation is repeated at an average interval of (T)/ false (F) and choose the correct option :
about 28/29 days
I. The menstrual phase is followed by the follicular
(b) The cycle of events starting from one menstruation phase
till the next one is called menstrual cycle
II. The changes in the ovary and uterus during
(c) One ovum is released (ovulation) during the menstrual cycle are induced by the changes in the
middle of each menstrual cycle levels of ovarian hormones only
(d) All of the above holds true III. Both LH and FSH attain a peak level in the middle
84. Identify the hormones (W, X, Y, Z) and the structures of cycle, i. e., about 24th day
(A, B) labelled in the diagram : IV. Rapid secretion of LH leading to its maximum
Hormone X
level during the mid-cycle called ‘LH-surge’
Hormone Y induces rupture of Graafian follicle and thereby
ovulation
Structure A Opti
Structure B I II III IV
ons

(a) F T T F

Hormone W (b) T F F T
Hormone Z
(c) T T F T

(d) F T T T

88. In females the ovulatory phase is followed by (w.r.t.


one menstrual cycle) :
(a) Luteal phase
(b) Follicular phase
Hor- Hor- Stru-
Opti Stru- Hor- Hor- (c) Secretion of excess estrogen by ovary and excess
mone mone cture
ons cture B mone W mone Z
X Y A LH by pituitary
(a) FSH LH Mature Corpus Estrogen Progesterone (d) More than one option
follicle luteum
89. Read the following statements :
LH FSH Mature Corpus Progesterone Estrogen
(b) follicle luteum (i) Corpus luteum secretes large amounts of
(c) FSH LH Mature Regressing Progesterone Estrogen progesterone which is essential for the
follicle corpus
luteum
maintenance of the endometrium
LH FSH Mature Corpus Estrogen Progesterone
(d) follicle luteum
Human Reproduction 811

(ii) The maintenance of endometrium is necessary 92. Graafian follicle forms :


for implantation of the fertilized ovum and other (a) Corpus luteum
events of pregnancy (b) Corpus callosum
(iii) During pregnancy all the events of the menstrual (c) Both (a) and (b)
cycle stop and there is menstruation (d) Secondary follicle
(iv) In the absence of fertilization, the corpus luteum 93. The phase with shortest duration in the menstrual cycle
degenerates which causes disintegration of is :
endometrium leading to menstruation, marking a (a) Ovulatory phase
new cycle
(b) Luteal phase
(v) Cyclic menstruation extends between menarche
(c) Follicular phase
and menopause
(d) Menstrual phase
How many of the above statements are correct ?
94. Just after ovulation, the mammalian egg is covered by
(a) Five (b) Four
an outer :
(c) Two (d) Three
(a) Corona radiata
90. Identify W, X, Y and Z in the schematic representation
(b) Plasma membrane
of menstrual cycle :
(c) Vitelline membrane
Cycle starts (d) Zona pellucida
28th day 1st day 95. Which of the following is correct for a typical menstrual
cycle ?
5th day (a) Endometrium regenerates between 5th–10th day
X (b) Rise in the level of progesterone occur after 15th day
(c) Both (a) and (b)
(d) Rise in the level of estrogen in the first 5 days
W 96. Which of the following phase of menstrual cycle in
human females lasts for about 7-8 days ?
Y (a) Ovulatory phase
(b) Menstruation
(c) Proliferative phase
(d) Secretory phase
14th day 97. If the menstrual cycle is of 31 days, then ovulation will
Z occur on :
Opti
(a) 14th day (b) 17th day
W X Y Z (c) 20th day (d) 12th day
ons
98. On which day from the start of menstruation the
(a) Menstrual Secretory Follicular Ovulation
phase phase phase chances of conception is maximum ?
(b) Luteal phase Secretory Ovulation Follicular (a) 14th day (b) 28th day
phase phase (c) 15thday (d) 5th day
(c) Luteal phase Menstrual Follicular Ovulation 99. Arrange the events of menstrual cycle in sequence :
phase phase I. LH surge
(d) Proliferative Menstrual Secretory Ovulation II. Regeneration of endometrium through
phase phase phase
proliferation
91. In humans, oogenesis is initiated during : III. Rupture of Graafian follicle
(a) Menarch IV. Regression of corpus luteum
(b) Menopause V. Menstruation
(c) Embryonic development VI. Secretions of gonadotropins and estrogen
(d) Fourteenth year from birth VII. Progesterone secretion
VIII. Corpus luteum formation
812 NCERT Biology Booster

(a) VI ® II ® I ® VII ® VIII ® III ® V ® IV


Pituitary B A
(b) VI ® II ® I ® III ® VIII ® VII ® IV ® V hormones
(c) I ® IV ® V ® VI ® III ® II ® VIII ® VII
(d) I ® II ® III ® VI ® VIII ® VII ® IV ® V C
Ovarian D
100. Which of the following is correct ?
hormones
(a) Estrogen controls the growth of uterine lining
(b) Decreased level of progesterone is the immediate
Days
cause of initiation of menstruation
(c) If egg is fertilized, increased level of progesterone (a) At menopause there is a rise in the urinary
stops ovulation during pregnancy excretion of B
(d) All of the above (b) D is secreted by corpus luteum
101. Match the columns : (c) B stimulates the secretion of C
Column-I Column-II (d) A which is steroidal causes ovulation

A. Menstrual phase (i) Day 15-28 Fertilisation and Implantation


B. Luteal phase (ii) Day 1-5 107. The enzymes that dissolve the membrane enveloping
C. Ovulation (iii) Day 1-13 the ovum and help sperm to enter the ovum are :
D. Follicular phase (iv) Day 14 (a) Sperm lysins (b) Inhibin
(c) Both (a) and (b) (d) Spectrin
(a) A = (ii), B = (iv), C = (i), D = (iii) 108. The hormone commonly used to test pregnancy is :
(b) A = (ii), B = (i), C = (iv), D = (iii) (a) LH (b) FSH
(c) A = (i), B = (iii), C = (ii), D = (iv) (c) hCG (d) Estrogen
(d) A = (ii), B = (i), C = (iii), D = (iv) 109. Identify A, B and C in the diagram :
102. In which phase the ‘LH-surge’ occurs during menstrual
cycle ?
(a) At the mid-cycle Sperm
(b) At the end of cycle
(c) At the luteal phase
(d) In the beginning of follicular phase
103. In human females, menstruation is repeated at an
average interval of about : C
(a) 32/34 days (b) 28/29 days A
(c) 38/33 days (d) 38/39 days
104. Choose the hormone that reaches the maximum level
during secretory/luteal phase of menstrual cycle :
(a) Progesterone (b) Estrogen B
(c) FSH and LH (d) LH
105. Choose the incorrect statement w.r.t. menstrual cycle : Opti
A B C
ons
(a) Between 12-14 days the peak level of LH and FSH
is attained (a) Zona pellucida Previtelline space Cells of corona
radiata
(b) The level of LH decreases 15th day onwards
(c) Menstrual flow lasts for 3-5 days (b) Previtelline Zona pellucida Cells of corona
space radiata
(d) During 14th day the level of estrogen decrease and
(c) Cells of corona Zona pellucida Previtelline space
progesterone increases radiata
106. Choose the incorrect statement for the hormones A, B, Vitelline Zona pellucida Cells of corona
(d)
C and D in graph shown. These hormones include membrane radiata
estrogen, lutenising hormone, progesterone and
follicle stimulating hormone :
Human Reproduction 813

110. Fertilisation occurs in : Opti


B E G H
(a) Cervix ons
(b) Uterus (a) Ovulation Fertilization Morula Blastocyst
(c) Ampullary-isthmic junction of fallopian tube
(b) Blastocyst Fertilization Morula Ovulation
(d) Any portion of fallopian tube implantation
111. Which of the following produces acrosome ? Fertilization Morula Blastocyst Blastocyst
(c)
(a) Golgi bodies implantation
(b) Mitochondria (d) Morula Blastocyst Fertilization Blastocyst
(c) Centrioles implantation
(d) Vacuoles
116. Identify A, B, X and Y in the diagram :
112. Capacitation of sperms is the :
(a) Maturation of sperms to render them competent to X
fertilize an ovum
(b) Inactivation of sperm outside the testis
(c) Capacity of sperms to increase the rate of ovulation Y
(d) Degeneration of sperm
A B
113. Ovum is released from the follicle in the :
(a) Primary oocyte stage Opti
A B X Y
(b) Secondary oocyte stage ons
(c) Either (a) or (b) (a) Morula Ovum Inner mass Trophoblast
(d) Fallopian tube in oogonial stage of cells layer
114. Which of the following is incorrect ? (b) Ovum Morula Inner mass Trophoblast
of cells layer
(a) Implantation of embryo occurs in morula stage
(c) Embryo Ovum Blastomere Trophoblast
(b) Fertilization is a physico-chemical process layer
(c) Fertilization occurs in fallopian tube Morula Blastocyst Inner mass Trophoblast
(d)
(d) After a sperm has penetrated an ovum, the entry of of cells layer
other sperms is prevented by the development of
fertilisation membrane 117. Identify A, B, C, D and E in the diagram showing human
115. Identify B, E, G and H in the diagram showing foetus within the uterus :
fertilization and passage of growing embryo through B
the fallopian tube :
C
D
A

E
814 NCERT Biology Booster

Opti 122. The physiological maturation of sperms with increased


A B C D E
ons motility and about-to-fertilizing capacity occurs in :
(a) Umbilical Placental Yolk sac Uterus Embryo (a) Vas deferens
cord villi cavity (b) Vagina
(b) Placental Umbilical Uterus Yolk sac Embryo (c) Epididymis
villi cord cavity
(d) Seminiferous tubule
(c) Umbilical Placental Uterus Yolk sac Embryo
cord villi cavity 123. If a sperm enters an ovum, the entry of other sperms is
(d) Umbilical Placental Uterus Yolk sac Zygote prevented by the :
cord villi cavity (a) Formation of fertilisation membrane
(b) Formation of vitelline membrane
118. Select the odd one out :
(c) Both (a) and (b)
(a) Ovum is released from the Graafian follicle in
secondary oocyte stage (d) Secretion of endolysin by sperms
(b) Sperm and ova fuse to form zygote Pregnancy and Embryonic Development
(c) Second meiotic division in secondary oocyte yields 124. Read the following statements :
unequal sized cells (i) Implantation occurs between 6-12 days of
(d) Meiotic division in the secondary oocyte forms the fertilization
second polar body and a haploid ootid (ii) Clevage starts while the egg is still in the fallopian
119. The sex of a baby (male/female) is determined by the : tube
(a) Father (iii) The resultant daughter cells of cleavage are
(b) Mother blastomeres
(c) Hormones of ovary (iv) Zona pellucida remains intact throughout the
(d) Hormones of testes cleavage divisions
120. Match the columns : (v) Cleavage results in a solid ball of cells called
morula with eight to sixteen cells
Column-I Column-II
How many of the above statements are correct ?
A. Acrosin (i) Dissolves corona radiata
(a) Five (b) Four
B. Corona penetrating (ii) Dissolves cementing (c) Two (d) Three
enzyme material between the
corona radiata cells 125. The mitotic division in zygote starts when it :
C. Hyaluronidase (iii) Digests zona pellucida (a) Moves through the isthmus of the oviduct
(b) Reaches the infundibulum of fallopian tube
(a) A = (iii), B = (ii), C = (i) (c) Implants in the ampulla region of fallopian tube
(b) A = (i), B = (ii), C = (iii) (d) Reaches cervix
(c) A = (ii), B = (i), C = (iii) 126. Arrange the events of embryo formation sequentially
(d) A = (iii), B = (i), C = (ii) and choose the correct option :
121. Arrange the events of fertilization sequentially and I. Blastocyst
choose the correct option : II. Mature embryo
I. Acrosomal reaction III. Morula
II. Karyogamy IV. Zygotic cleavage
III. Entry of sperm into egg cytoplasm V. Blastomere
IV. Completion of meiosis-II in ovum (a) IV ® III ® I ® V ® II
V. Zygote formation (b) V ® II ® I ® III ® IV
(a) I ® II ® III ® V ® IV (c) IV ® III ® V ® I ® II
(b) II ® I ® III ® V ® IV (d) IV ® V ® III ® I ® II
(c) I ® III ® II ® IV ® V
(d) I ® III ® IV ® II ® V
Human Reproduction 815

127. Find the true (T)/ false (F) statements and choose the (b) Chorionic villi alone
correct option w.r.t. pregnancy : (c) Interdigitation of chorionic villi and uterine tissue
I. The trophoblast layer of blastocyst gets attached (d) Vaginal tissue
to the endometrium and inner cell mass 133. The structural and functional unit between the
differentiates into embryo developing foetus and maternal body is called :
II. After the attachment of blastocyst to the (a) Yolk sac (b) Cervix
endometrium, the uterine cells divide rapidly and (c) Vagina (d) Placenta
covers the blastocyst 134. Arrange the sequence of embryonic development and
III. Blastocyst finally gets embedded in the choose the correct option :
endometrium of the uterus which is called I. Morula
implantation II. Zygote
IV. After implantation embryo gets nutrition from III. Gastrula
yolk sac
IV. Blastula
Opti V. Foetus
I II III IV
ons (a) II ® I ® IV ® III ® V
(a) T T T F (b) II ® I ® III ® IV ® V
(c) II ® III ® I ® IV ® V
(b) T T F F
(d) II ® V ® IV ® III ® I
(c) T F T F 135. The placenta :
(d) T F F T (a) Is connected to the embryo through an umbilical
cord
128. Gastrulation : (b) Acts as endocrine tissue which produces hCG, hPL,
(a) Is the early phase in the embryonic development estrogens and progesterone
during which the blastula is reorganised into (c) Facilitates the supply of oxygen and nutrients to
multilayered gastrula the embryo and also removal of carbon-di-oxide
(b) Forms gastrula containing only one germ layer and excretory/waste materials produced by the
(c) Both (a) and (b) embryo
(d) Forms blastocyst (d) Is related to all of the above
129. Progesterone is produced by : 136. How many of the hormones given below are produced
(a) Corpus luteum in women only during pregnancy ?
(b) Placenta Estrogen, Human chorionic gonadotropin (hCG),
(c) Adrenal glands Human placental lactogen (hPL), Relaxin, Progesterone
(d) More than one
130. Choose the incorrect statement : (a) Four (b) Two
(a) Cleavage of zygote leads to increased mass of (c) Three (d) One
protoplasm 137. Read the following statements :
(b) Both monozygotic and dizygotic twins are (i) During pregnancy, the levels of thyroxin,
produced from single ovum prolactin, cortisol, estrogen and progesterone
(c) Increasing levels of progesterone causes uterine decrease
contraction (ii) Progesterone in increased level is essential for
(d) More than one option supporting the foetal growth and maintenance of
131. After implantation, finger-like projections appear on pregnancy
the trophoblast called : (iii) High levels of LH, FSH in the uterus and high
(a) Chorionic villi (b) Intestinal villi levels of circulating hCG stimulates the
(c) Placental villi (d) All of these endometrial thickening
132. Placenta is formed by the : (iv) High levels of LH and FSH in the uterus
(a) Uterine tissue alone stimulates implantation
816 NCERT Biology Booster

(v) High levels of hCG stimulates progesterone 143. Arrange the foetal membrane from outside to inside :
synthesis I. Amnion
Which of the above statements are correct ? II. Allantois
(a) (i), (ii) (b) (ii), (v) III. Chorion
(c) (iii), (iv) (d)(i), (iii) (a) III ® II ® I (b) III ® I ® II
138. Choose the odd one out w.r.t. placental hormone : (c) II ® I ® III (d) I ® II ® III
(a) hCG and hPL 144. During human embryonic development, which organ is
(b) Estrogen and progesterone formed first ?
(c) Relaxin and prolactin (a) Neural tube (b) Brain
(d) Human placental lactogen and progesterone (c) Skin (d) Heart
139. Choose the correct statement : 145. Match the column w.r.t. germ layers :
(a) After about nine weeks, the embryo is called a
Column-I Column-II
foetus
A. Ectoderm (i) Muscles, skeleton, heart,
(b) Gestation is the carrying of embryo/foetus inside
kidney, gonads
the female body
B. Mesoderm (ii) Nervous system, skin
(c) In humans, birth normally occurs at a gestational C. Endoderm (iii) Respiratory organs and
period of about 37-42 weeks mid-gut
(d) All of these
140. The inner cell mass of a blastocyst :
(a) A = (i), B = (ii), C = (iii)
(a) Later forms ectoderm and endoderm but no
(b) A = (ii), B = (i), C = (iii)
mesoderm
(c) A = (iii), B = (ii), C = (i)
(b) Contain stem cells
(d) A = (ii), B = (iii), C = (i)
(c) Degenerates during foetus development
(d) Both (a) and (c) Parturition and Lactation
141. Match the columns w.r.t. to months/trimester and 146. What do the doctors inject to induce labor ?
human embryo development : (a) Oxytocin (b) Progesterone
(c) LH (d) FSH
Column-I Column-II
147. Match the columns :
A. End of 1st month (i) Limbs and genital
organs become well Column-I Column-II
developed
A. Parturition (i) Milk production
B. End of 2nd month (ii) Body hair develops,
eyelids separate and B. Lactation (ii) Initial milk with several
eyelashes are formed antibodies, healthy protein
C. End of 1st trimester (iii) Heart is formed and less fat
D. End of 2nd trimester (iv) Limbs and digits C. Colostrum (iii) Foetus delivery
develops D. Oxytocin (iv) Uterine contraction

(a) A = (i), B = (iv), C = (ii), D = (iii) (a) A = (iv), B = (i), C = (ii), D = (iii)
(b) A = (iii), B = (iv), C = (ii), D = (i) (b) A = (ii), B = (iv), C = (i), D = (iii)
(c) A = (iii), B = (iv), C = (i), D = (ii) (c) A = (iii), B = (ii), C = (iv), D = (i)
(d) A = (ii), B = (i), C = (iii), D = (iv) (d) A = (iii), B = (i), C = (ii), D = (iv)
142. The first movement of the foetus and appearance of 148. The foetal ejection-reflex in human females is induced
hair on the head are usually observed during which by :
month of pregnancy ? (a) Fully developed foetus and placenta
(a) 3rd month (b) Release of oxytocin from the anterior pituitary
(b) 9th month (c) Amniotic fluid
(c) 2nd month (d) Lutenising hormone
(d) 5th month
Human Reproduction 817

Miscellaneous Opti
W X Y Z
149. Identify X, M, N, O and P for the hormonal control of ons
male reproductive system : (a) GnRH FSH/LH Progesterone Estrogen
– (b) GnRH FSH/LH Estrogen Progesterone
Hypothalamus
(c) FSH/LH GnRH Estrogen Progesterone
Hormone X
(d) FSH/LH GnRH Progesterone Estrogen
– –
Anterior pituitary
152. The cross at the mid region of the middle piece of a
human spermatozoan will reveal :
LH FSH (a) ‘9 + 2’ arrangement of microtubules and numerous
TESTES mitochondria
M N (b) Centrioles
+ (c) Mitochondria only
Hormone + (d) Mitochondria and centriole
O Stimulates
spermatogenesis
153. Match the columns :

+ + Column-I Column-II
Secondary Hormone A. Endometrium (i) Zona pellucida
sex organs P
B. Clitoris (ii) Alveoli
Opti C. Mammary gland (iii) Erogenous zone
X M N O P
ons D. Secondary oocyte (iv) Womb
(a) GnRH Leydig cells Sertoli Androgens Inhibin
cells (testosterone) (a) A = (iv), B = (iii), C = (i), D = (ii)
(b) Inhibin Leydig cells Sertoli Androgens GnRH (b) A = (ii), B = (iv), C = (i), D = (iii)
cells
(c) A = (iii), B = (i), C = (iv), D = (ii)
(c) Androgens Leydig cells Sertoli GnRH Inhibin
cells
(d) A = (iv), B = (iii), C = (ii), D = (i)
GnRH Sertoli cells Leydig Androgens Inhibin 154. The layers of ovum from inside to outside is :
(d)
cells (a) Corona radiata ® Vitelline membrane ® Zona
pellucida
150. A high rate of metabolic activity is maintained in the
(b) Vitelline membrane ® Zona pellucida ® Corona
pregnant uterus by :
radiata
(a) Testosterone (b) Inhibin and oxytocin
(c) Corona radiata ® Zona pellucida ® Vitelline
(c) Progesterone (d) Prolactin
membrane
151. Identify the hormones W, X, Y and Z for the hormonal
(d) Zona pellucida ® Corona radiata ® Vitelline
control of female reproductive system :
membrane
+ – – 155. What is the effect of menopause on the levels of FSH,
Hypothalamus
LH, estrogen and GnRH ?
Hormone W (a) [ FSH] ­; [ LH] ­; [ Estrogen] ¯; [GnRH] ­
(b) [ FSH] ¯; [ LH] ¯; [ Estrogen] ¯; [GnRH] ¯
+ – –
Anterior pituitary (c) [ FSH] ¯; [ LH] ¯; [ Estrogen] ­; [GnRH] ­
(d) [ FSH] ¯; [ LH] ­; [ Estrogen] ¯; [GnRH] ¯
Hormone X
156. Which of the following is incorrect for estrous cycle ?
Ovary
(a) It occurs in all the primates
(b) There is no menstruation at the end of estrous cycle
(c) The estrogen level in blood increases, resulting in
‘period of heat’
Hormone Y
Hormone Z (d) It remains suspended during non-breeding season
818 NCERT Biology Booster

157. The age of menopause in human females is :


(a) 75 years (b) 32 years
(c) 45-50 years (d) 28 years

Answers (Section-A)

1. (b) 2. (c) 3. (d) 4. (d) 5. (a) 6. (d) 7. (a) 8. (d) 9. (d) 10. (d)
11. (c) 12. (a) 13. (d) 14. (a) 15. (c) 16. (d) 17. (d) 18. (c) 19. (a) 20. (b)
21. (a) 22. (a) 23. (d) 24. (a) 25. (c) 26. (b) 27. (a) 28. (c) 29. (a) 30. (a)
31. (a) 32. (a) 33. (a) 34. (c) 35. (d) 36. (d) 37. (b) 38. (d) 39. (b) 40. (d)
41. (c) 42. (b) 43. (b) 44. (c) 45. (d) 46. (d) 47. (b) 48. (c) 49. (b) 50. (b)
51. (d) 52. (c) 53. (b) 54. (c) 55. (b) 56. (d) 57. (c) 58. (a) 59. (d) 60. (b)
61. (c) 62. (b) 63. (d) 64. (a) 65. (a) 66. (c) 67. (c) 68. (d) 69. (d) 70. (a)
71. (b) 72. (d) 73. (d) 74. (c) 75. (d) 76. (c) 77. (c) 78. (a) 79. (a) 80. (a)
81. (b) 82. (a) 83. (d) 84. (d) 85. (d) 86. (c) 87. (b) 88. (a) 89. (b) 90. (c)
91. (c) 92. (a) 93. (a) 94. (a) 95. (c) 96. (c) 97. (b) 98. (a) 99. (b) 100. (d)
101. (b) 102. (a) 103. (b) 104. (a) 105. (d) 106. (d) 107. (a) 108. (c) 109. (a) 110. (c)
111. (a) 112. (a) 113. (b) 114. (a) 115. (c) 116. (d) 117. (c) 118. (b) 119. (a) 120. (d)
121. (d) 122. (b) 123. (a) 124. (a) 125. (a) 126. (d) 127. (a) 128. (c) 129. (d) 130. (d)
131. (a) 132. (c) 133. (d) 134. (a) 135. (d) 136. (c) 137. (b) 138. (c) 139. (d) 140. (b)
141. (c) 142. (d) 143. (b) 144. (a) 145. (b) 146. (a) 147. (d) 148. (a) 149. (a) 150. (c)
151. (b) 152. (a) 153. (d) 154. (b) 155. (a) 156. (a) 157. (c)
Human Reproduction 819

Section-B : Questions from NCERT Exemplar

1. Choose the incorrect statement from the following : 7. Acrosomal reaction of the sperm occurs due to :
(a) In birds and mammals internal fertilization takes (a) its contact with zona pellucida of the ova
place (b) reactions within the uterine environment of the
(b) Colostrum contains antibodies and nutrients female
(c) Polyspermy in mammals is prevented by the (c) reactions within the epididymal environment of
chemical changes in the egg surface the female
(d) In the human female implantation occurs almost (d) androgens produced in the uterus
seven days after fertilization.
8. Which one of the following is not a male accessory
2. Identify the correct statement from the following : gland ?
(a) High levels of estrogen triggers the ovulatory (a) Seminal vesicle (b) Ampulla
surge
(c) Prostate (d) Bulbourethral gland
(b) Oogonial cells start to proliferate and give rise to
9. The immature male germ cell undergo division to
functional ova in regular cycles from puberty
produce sperms by the process of spermatogenesis.
onwards
Choose the correct one with reference to above :
(c) Sperms released from seminiferous tubules are (a) Spermatogonia have 46 chromosomes and always
highly motile. undergo meiotic cell division
(d) Progesterone level is high during the post (b) Primary spermatocytes divide by mitotic cell
ovulatory phase of menstrual cycle. division
3. Spot the odd one out from the following structures (c) Secondary spermatocytes have 23 chromosomes
with reference to the male reproductive system : and undergo second meiotic division
(a) Rete testis (b) Epididymis (d) Spermatozoa are transformed into spermatids.
(c) Vasa efferentia (d) Isthmus 10. Match between the following representing parts of the
4. Seminal plasma, the fluid part of semen, is contributed sperm and their functions and choose the correct
by : option :
(i) Seminal vesicle (ii) Prostate
Column-I Column-II
(iii) Urethra (iv) Bulbourethral gland
A. Head (i) Enzyme
(a) (i) and (ii) (b) (i), (ii) and (iv)
B. Middle piece (ii) Sperm motility
(c) (ii), (iii) and (iv) (d) (i) and (iv)
C. Acrosome (iii) Energy
5. Spermiation is the process of the release of sperms
from : D. Tail (iv) Genetic material
(a) Seminiferous tubules
(a) A = (ii), B = (iv), C = (i), D = (iii)
(b) Vas deferens
(b) A = (iv), B = (iii), C = (i), D = (ii)
(c) Epididymis
(c) A = (iv), B = (i), C = (ii), D = (iii)
(d) Prostate gland
(d) A = (ii), B = (i), C = (iii), D = (iv)
6. Mature Graafian follicle is generally present in the
11. Which among the following has 23 chromosomes ?
ovary of a healthy human female around :
(a) Spermatogonia (b) Zygote
(a) 5-8 day of menstrual cycle
(c) Secondary oocyte (d) Oogonia
(b) 11-14 day of menstrual cycle
12. Urethral meatus refers to the :
(c) 18-23 day of menstrual cycle
(a) Urinogenital duct
(d) 24-28 day of menstrual cycle (b) Opening of vas deferens into urethra
820 NCERT Biology Booster

(c) External opening of the urinogenital duct 15. The vas deferens receives duct from the seminal vesicle
(d) Muscles surrounding the urinogenital duct and opens into urethra as :
13. Match the following and choose the correct option : (a) epididymis (b) ejaculatory duct
(c) efferent ductule (d) ureter
A. Trophoblast (i) Embedding of blastocyst in the
endometrium 16. Morula is a developmental stage :
B. Cleavage (ii) Group of cells that would
(a) between the zygote and blastocyst
differentiate as embryo (b) between the blastocyst and gastrula
C. Inner cell mass (iii) Outer layer of blastocyst (c) after the implantation
attached to the endometrium (d) between implantation and parturition
D. Implantation (iv) Mitotic division of zygote 17. The membranous cover of the ovum at ovulation is :
(a) corona radiata (b) zona radiata
(a) A = (ii), B = (i), C = (iii), D = (iv)
(c) zona pellucida (d) chorion
(b) A = (iii), B = (iv), C = (ii), D = (i)
18. Identify the odd one from the following :
(c) A = (iii), B = (i), C = (ii), D = (iv)
(a) Labia minora (b) Fimbriae
(d) A = (ii), B = (iv), C = (iii), D = (i)
(c) Infundibulum (d) Isthmus
14. Which of the following hormones is not secreted by
human placenta ?
(a) hCG (b) Estrogens
(c) Progesterone (d) LH

Answers (Section-B)
1. (c) 2. (d) 3. (d) 4. (b) 5. (a) 6. (b) 7. (a) 8. (b) 9. (c) 10. (b)
11. (c) 12. (c) 13. (b) 14. (d) 15. (b) 16. (a) 17. (a) 18. (a)
Human Reproduction 821

Section-C : Assertion-Reason Type Questions


Instructions :
1. If both assertion and reason are true and the reason is the correct explanation of the assertion then mark (a).
2. If both assertion and reason are true but the reason is not the correct explanation of the assertion, then mark (b).
3. If assertion is true but reason is false, then mark (c).
4. If both assertion and reason are false statements then mark (d).

1. A: Holoblastic cleavage with almost equal sized 10. A: Soon after the infant is delivered, the placenta is
blastomeres is a characteristics of placental also expelled out of the uterus.
animals. R: Expulsion of placenta is called parturition.
R: Eggs of most mammals including humans are of 11. A: Blood flowing in umbilical cord of human embryo is
centrolecithal type. 100% foetal.
2. A: Parturition is peristalsis expulsion of foetus through R: Umbilical cord helps in transport of substances
birth canal. between foetal and maternal blood.
R: Oxytocin stimulates peristalsis in uterus and also 12. A: The seminal plasma is rich in fructose, calcium and
ejection of milk. certain enzymes.
3. A: Corpus luteum are cells produced by Graafian R: Bulbourethral gland does not contribute anything
follicle after ovulation. to seminal plasma.
R: It secretes estrogen, which is necessary to maintain 13. A: Menstrual cycle stops during pregnancy.
pregnancy. R: Stoppage of menstrual cycle confirms pregnancy.
4. A: The endometrium of a 35 years old pregnant lady 14. A: The menstrual flow occurs due to the breakdown of
fails to degenerate. endometrial lining of uterus and its blood vessels.
R: Progesterone concentration increases due to R: Rapid fall in the level of progesterone takes place
fertilisation. due to degeneration of corpus luteum.
5. A: Corpus luteum start degenerating after fertilisation. 15. A: hCG, and hPL are produced in women only during
R: Corpus luteum secretes hormone responsible for pregnancy.
ovulation. R: These hormones are produced by placenta.
6. A: All copulations do not lead to fertilisation and 16. A: As a result of ovariectomy, menstrual cycle may
pregnancy. stop.
R: Fertilisation can occur only if the ovum and sperms R: Ovarian hormones induce menstrual cycle.
are transported simultaneously to ampullary- 17. A: The inner cell mass of blastocyst differentiates into
isthmic junction. outer and inner layer just before implantation.
7. A: Only one sperm can fertilise an ovum. R: The outer layer of blastocyst forms the embryo.
R: During fertilisation, a sperm comes in contact with 18. A: In a menstrual cycle secretory phase is followed by
the zona pellucida layer of ovum and induces follicular phase.
changes in the membrane that blocks the entry of R: Secretory phase is called follicular phase.
additional sperms. 19. A: Ootid is haploid.
8. A: During early pregnancy, hCG prevents the R: Ovum is haploid.
disintegration of corpus luteum.
20. A: Parturition is induced by a complex neuroendocrine
R: Under the influence of hCG, the corpus luteum mechanism involving cortisol, estrogen and
continues to produce progesterone which keeps oxytocin.
endometrium intact.
R: The signals for parturition originate from fully
9. A: A pregnant woman cannot concieve again during developed foetus and placenta which induce foetal
pregnancy. ejection reflex.
R: Corpus luteum secretes high level of progesterone
and estrogen during pregnancy which inhibits the
secretion of gonadotropins and thereby prevent
ovulation.
822 NCERT Biology Booster

Answers (Section-C)
1. (c) 2. (b) 3. (c) 4. (a) 5. (d) 6. (a) 7. (a) 8. (b) 9. (a) 10. (c)
11. (b) 12. (c) 13. (c) 14. (a) 15. (a) 16. (a) 17. (d) 18. (d) 19. (b) 20. (b)
Human Reproduction 823

Section-D : Brainstormer
1. In eutherians, the placenta is formed from : (iv) Progesterone production is largely under the
(a) Allantois and amnion control of LH
(b) Amnion and chorion (v) Throughout the part of menstrual cycle that
follows ovulation, there is slight rise in body
(c) Chorion and allantois temperature
(d) Amnion and yolk sac How many of the above statements are correct ?
2. How are the time of ovulation and the onset of (a) Two (b) Three
menstruation related in human menstrual cycle? (c) Four (d) One
(a) Both are triggered by high luteinizing hormone 6. How does menstrual cycle become initiated when
(LH) spikes, i. e., sharp increase fertilization fails to occur ?
(b) Ovulation is triggered by copulation, menstruation (a) The nervous system detects the absence of
is triggered by hormonal effect implantation, signalling the relase of GnRH from
(c) Ovulation is triggered by harmonal effect and the hypothalamus, which triggers the release of
menstruation is triggered by copulation FSH from pituitary
(d) Ovulation occurs approximately 14 days before the (b) hCG produced by the uterine lining signals the
first day of menstruation corpus luteum to regress, releasing the
3. During the first two weeks of the human menstrual hypothalamus and pituitary from feedback
cycle, artificially increasing blood levels of estrogen inhibition. FSH level rise.
and progesterone will : (c) Progesterone level drop due to regression of the
(a) Cause menstruation to start early corpus luteum, releasing the hypothalamus and
(b) Inhibit ovulation by reducing the release of LH and pituitary from feedback inhibition. FSH levels rise.
FSH from the pituitary (d) hCG produced by the embryo following
(c) Cause development of the ovarian follicle through implantation in the uterus, signals the corpus
binding to its receptors luteum to regress. Progesterone level drops and
(d) Stimulate secretion of milk by the lacteal glands in FSH level rise.
the breasts 7. Oxytocin shows following consequences in the human
4. Trace the passage of a sperm cell from the structure body:
where it is produced till fertilization of the egg. I. Uterine contraction
I. Semniferous tubule II. Milk ejection
II. Vasa deferens III. Antidiuresis
III. Uterus The results are due to the effect on :
IV. Fallopian tube (a) Voluntary and involuntary muscles
V. Vagina (b) Smooth and striated muscles
VI. Epididymis (c) Striated muscles and membrane depolarisation
VII. Urethra (d) Smooth muscles and membrane permeability
(a) VI ® I ® II ® VII ® V ® II ® IV
8. In humans, the unpaired male reproductive structure
(b) I ® VI ® II ® VII ® V ® III ® IV is:
(c) I ® VI ® II ® VII ® V ® IV ® III (a) Prostate (b) Bulbourethral gland
(d) I ® II ® VI ® VII ® V ® III ® IV (c) Seminal vesicle (d) Vasa deferens
5. Read the following statements : 9. In human female, the blastocyst :
(i) Both estrogen and progesterone are necessary (a) Gets nutrition from uterine endometrial secretions
for ovulation to take place only after implantation
(ii) Estrogen tends to inhibit the secretion of FSH (b) Gets implanted in endometrium by the trophoblast
from the anterior pituitary gland
cell
(iii) Fertilization of the ovum by the spermatozoan
(c) Gets implanted into uterus three days after
normally takes place in the uterus
ovulation
824 NCERT Biology Booster

(d) Forms placenta even before implantation (c) Zona pellucida remains intact throughout the
10. Choose the wrong statement : cleavage divisions
(a) During cleavage, there is increase in the mass of (d) During cleavage, there is no increase in the mass of
cytoplasm of developing embryo but the DNA cytoplasm of developing embryo but increase in
content remains constant DNA content occurs
(b) During cleavage, the size of blastomeres keep on
decreasing

Answers (Section-D)
1. (c) 2. (d) 3. (b) 4. (b) 5. (a) 6. (c) 7. (d) 8. (a) 9. (b) 10. (a)
Human Reproduction 825

Section-E : Latest Previous Years Questions

1. Which one of the following events is correctly matched (c) During normal menstruation about 40 mL blood is
with the time period in a normal menstrual cycle? lost
(a) Release of egg : 5th day (d) The menstrual fluid can easily clot
(b) Endometrium regenerates : 5-10 days 8. The correct sequence of spermatogenetic stages
(c) Endometrium secretes nutrients for implantation: leading to the formation of sperms in a mature human
11 - 18 days testis is :
(d) Rise in progesterone level : 1-15 days (a) Spermatogonia - Spermatid - Spermatocyte -
Sperms
2. If mammalian ovum fails to get fertilized, which one of
the following is unlikely ? (b) Spermatocyte - Spermatogonia - Spermatid -
Sperms
(a) Estrogen secretion further decreases
(c) Spermatogonia - Spermatocyte - Spermatid -
(b) Progesterone secretion rapidly declines
Sperms
(c) Corpus luteum will disintegrate
(d) Spermatid - Spermatocyte - Spermatogonia -
(d) Primary follicle starts developing Sperms
3. Withdrawl of which of the following hormones is the 9. Which one of the following in the most likely root
immediate cause of menstruation : cause why menstruation is not taking place in regularly
(a) Estrogen (b) FSH cycling human female ?
(c) FSH-LH (d) Progesterone (a) Retention of well-developed corpus luteum
4. Sertoli cells are regulated by the pituitary hormone (b) Fertilisation of the ovum
known as : (c) Maintenance of the hypertrophical endometrial
(a) FSH (b) GH lining
(c) Prolactin (d) LH (d) Maintenance of high concentration of sex
hormones in the blood stream
5. Compared to a bull, a bullock is docile because of :
10. Which one of the following is the correct matching of
(a) Lower leavels of adrenalin/ noradrenalin is its
blood the events occurring during menstrual cycle ?
(a) Menstruation : Breakdown of myometrium
(b) Higher levels of thyroxin
and ovum not fertilised
(c) Higher levels of cortisone
(b) Ovulation : LH and FSH attain peak
(d) Lower levels of blood testosterone level and sharp fall in the
6. In the human female, menstruation can be deferred by secretion of progesterone.
the administration of: (c) Proliferative : Rapid regeneration of
(a) FSH only phase myometrium and maturation
(b) LH only of Graafian follicle.
(c) Combination of FSH and LH (d) Development of : Secretory phase and incresed
corpus luteum secretion of progesterone.
(d) Combination of estrogen and progesterone
11. Foetal ejection reflex in human female is induced by :
7. Which one of the following statements is incorrect
(a) Differentiation of mammary glands
about menstruation ?
(b) Pressure exerted by amniotic fluid
(a) At menopause in the female, there is especially
abrupt increase in gonadotropic hormone (c) Release of oxytocin from pituitary
(b) The beginning of the cycle of menstruation is (d) Fully developed foetus and placenta
called menarche 12. A change in the amount of yolk and its distribution in
the egg will affect :
826 NCERT Biology Booster

(a) Fertilization (b) It has almost equal quantity of cytoplasm as an


(b) Formation of zygote uncleaved zygote but much more DNA
(c) Pattern of cleavage (c) It has far less cytoplasm as well as less DNA than in
(d) Number of blastomeres produced an uncleaved zygote
(d) It has more or less equal quantity of cytoplasm and
13. Given below is a diagrammatic sketch of a portion of
DNA as in uncleaved zygote
human male reproductive system. Select the correct set
of the names of the parts labelled A, B, C D : 17. Sertoli cells are found in :
(a) Pancreas and secrete cholecystokinin
(b) Ovaries and secrete progesterone
A (c) Adrenal cortex and secrete adrenaline
(d) Seminiferous tubules and provide nutrition to
B
germ cells
C 18. The part of fallopian tube closest to the ovary is :
D
(a) Ampulla (b) Isthmus
(c) Infundibulum (d) Cervix
19. If for some reason, the vasa efferentia in the human
(A) (B) (C) (D) reproductive system get blocked, the gametes will not
be transported from :
(a) Ureter Seminal Prostate Bulbourethral
vesicle gland (a) Testes to epididymis
(b) Epididymis to vas deferens
(b) Ureter Prostate Seminal Bulbourethral
vesicle gland (c) Ovary to uterus
(c) Vas Seminal Prostate Bulbourethral (d) Vagina to uterus
deferens vesicle gland 20. Which one of the following statements about human
(d) Vas Seminal Bulbourethral Prostate sperms is correct ?
deferens vesicle gland (a) Acrosome serves no particular function
(b) Acrosome has a conical pointed structure in the
14. Seminal plasma in humans is rich in :
egg, resulting in fertilisation
(a) Fructose and certain enzymes but poor in calcium
(c) The sperm lysins in the acrosome dissolve the egg
(b) Fructose, calcium and certain enzymes envelope facilitating fertilisation
(c) Fructose and calcium but has no enzymes (d) Acrosome serves as a sensory structure leading the
(d) Glucose and certain enzyme sperm towards the ovum
15. The second maturation division of the mammalian 21. Vasa efferentia are the ductuless leading from:
ovum occurs: (a) Epididymis to urethra
(a) In the Graafian follicle following the first (b) Testicular lobules to rete testis
maturation division
(c) Rete testis to vas deferens
(b) Shortly after ovulation before the ovum makes
(d) Vas deferens to epididymis
entry into the Fallopian tube
22. The first movements of the foetus and appearance of
(c) After the ovum has been penetrated by a sperm
hair on its head are usually observed during which
(d) After the nucleus of the sperm has fused with that
month of pregnancy ?
of the ovum
(a) Third month (b) Fourth month
16. Which one of the following statements about morula in
(c) Fifth month (d) Sixth month
humans is correct ?
23. Signals from fully developed foetus and placenta
(a) It has more cytoplasm and more DNA that an
uncleaved zygote ultimately lead to parturition which requires the
release of :
Human Reproduction 827

(a) Oxytocin from maternal pituitary


(b) Oxytocin from foetal pituitary
(c) Relaxin from placenta
(d) Estrogen from placenta III
I IV
24. In human female the blastocyst:
II V
(a) Gets implanted into uterus 3 days after ovulation
(b) Gets nutrition form uterine endometrial secretions
only after implantation VI

(c) Gets implanted in endometrium by the


(a) (II) Endometrium, (III) Infundibulum, (IV)
trophoblast cells
Fimbriae
(d) Forms placenta even before implantation
(b) (III) Infundibulum, (IV) Fimbriae, (V) Cervix
25. In the given diagram identify A, B, C and D respectively
(c) (IV) Oviducal funnel, (V) Uterus. (VI) Cervix
and choose the correct option out of 1, 2, 3 and 4 :
(d) (I) Perimetrium, (II) Myometrium, (III) Fallopian
C tube
B
28. The testes in humans are situated outside the
abdominal cavity inside a pouch called scrotum. The
A
purpose served is for :
(a) Maintaining the scrotal temperature lower than
D
the internal body temperature
(b) Escaping any possible compression by the visceral
organs.
(c) Providing more space for the growth of epididymis
(A) (B) (C) (D) (d) Providing a secondary sexual feature for
exhibiting the male sex.
(a) Fertilization Mitosis Implantation Uterus
29. About which day in normal human menstrual cycle
(b) Implantation Mitosis-II Fertilization Uterus does rapid secretion of LH (popularly called LH-surge)
(c) Fertilization Implantation Mitosis Uterus normally occurs :
(d) Mitosis Fertilization Implantation Uterus (a) 11 th day (b) 14 th day
(c) 20 th day (d) 5 th day
26. Write the correct sequence of given diagram of 30. What happens during fertilisation in humans after
developmental events : many sperms reach close to the ovum ?
(a) Only two sperms nearest the ovum penetrate zona
pellucida
(b) Secretion of acrosome helps one sperm enter
cytoplasm of ovum through zona pellucida
(c) All sperms except the one nearest to the ovum lose
(A) (B) (C) (D)
their tails
(a) A, B, C, D (b) B, C, A, D (d) Cells of corona radiata trap all the sperm except
(c) C, B, A, D (d) C, A, B, D one
27. The figure given below depicts a diagrammatic 31. From A to D which of the following statements are
sectional view of the female reproductive system of correct :
human. Which one set of three parts out of I-VI have A. Fertilisation can occur only if the ovum and sperm
been correctly identified ? are transported simultaneously to the ‘ampullary
isthmic junction’
828 NCERT Biology Booster

B. First movement of foetus and appearance of hair


on head are usually observed during the fifth
month of pregnancy
C. On 14 th day of menstrual cycle, the level of
progesterone is maximum.
D. The signals for parturition originate from fully
developed foetus and placenta Developmental
Site of occurrence
stage
(a) A and C (b) B and C
(c) C and D (d) B and D (a) Blastocyst Uterine wall

32. In a normal pregnant woman, the amount of total (b) 8-celled morula Starting point of Fallopian tube
gonadotropin actively was assessed. The result (c) Late morula Middle part of Fallopian tube
expected was :
(d) Blastula End part of Fallopian tube
(a) High levels of FSH and LH in uterus to stimulate
endometrial thickening 37. What is the correct sequence of sperm formation ?
(b) High levels of circulating hCG to stimulate (a) Spermatogonia, Spermatocyte, Spermatid,
estrogen and progesterone synthesis Spermatozoa
(c) High levels of circulating FSH and LH in the uterus (b) Spermatid, Spermatocyte, Spermatogonia,
to stimulate implantation of the embryo Spermatozoa
(d) High level of circulating hCG to stimulate (c) Spermatogonia, Spermatocyte,Spermatozoa,
endometrial thickening Spermatid
33. Signals for parturition originate from : (d) Spermatogonia, Spermatozoa, Spermatocyte,
(a) Placenta only Spermatid
(b) Fully developed foetus only 38. Menstrual flow occurs due to lack of :
(c) Both placenta as well as fully developed foetus (a) Vasopressin (b) Progesterone
(d) Oxytocin released from maternal pituitary
(c) FSH (d) Oxytocin
34. Which one of the following statements is false in
39. Which one of the following is not the function of
respect of viability of mammalian sperm ?
placenta ? It :
(a) Viability of sperm is determined by its motility
(a) Secretes oxytocin during parturition
(b) Sperms must be concentrated in a thick
suspension (b) Facilitates supply of oxygen and nutrients to
embyro
(c) Sperm is viable for only up to 24 hours
(c) Secretes estrogen
(d) Survival of sperm depends on this pH of the
medium and is more active in alkaline medium (d) Facilitates removal of carbon dioxide and waste
material from embryo
35. The leydig cells as found in the human body are the
secretory source of : 40. Correct sequence of male accessory glands pouring
(a) glucagon their secretions into urethra :
(b) androgens (a) Prostate, Seminal vesicle, Bulbourethral glands
(c) progesterone (b) Seminal vesicle, Prostate, Bulbourethral glands
(d) intestinal mucus (c) Bulbourethral glands, Prostate, Seminal vesicle
36. Identify the human development stage shown below as (d) Prostate, Bulbourethral glands, Seminal vesicle
well as the related right place of its occurrence in a 41. The shared terminal duct of the reproductive and
normal pregnant woman, and select the right option urinary system in the human male is :
for together :
(a) Urethra (b) Ureter
(c) Vas deferens (d) Vasa efferentia
Human Reproduction 829

42. The main function of mammalian corpus luteum is to (d) the sperms are transported into cervix within 48
produce : hrs of release of ovum in uterus
(a) estrogen only 49. Select the incorrect statement :
(b) progesterone (a) FSH stimulates the sertoli cells which help in
spermiogenesis
(c) human chorionic gonadotropin
(b) LH triggers ovulation in ovary
(d) relaxin only
(c) LH and FSH decrease gradually during the
43. Select the correct option describing gonadotropin follicular phase
activity in a normal pregnant female: (d) LH triggers secretion of androgens from the Leydig
(a) High levels of FSH and LH stimulates the cells
thickening of endometrial thickening
50. Identify the correct statement on inhibin :
(b) High levels FSH and LH facilitate implantation of (a) Inhibits the secretion of LH, FSH and Prolactin
the embryo
(b) Is produced by granulosa cells in ovary and
(c) High levels of hCG stimulates the synthesis of inhibits the secretion of FSH
estrogen and progesterone. (c) Is produced by granulosa cells in ovary and
(d) High level of hCG stimulate the thickening inhibits the secretion of LH
endometrium (d) Is produced by nurse cells in testes and inhibits the
44. LH and FSH level during human menstrual cycle secretion of LH
increase on : 51. Capacitation occurs in :
(a) 14th day (b) 7th day (a) Epididymis
(c) 21 day (d) 9th day (b) Vas deferens
45. Capacitation refers to changes in the : (c) Female reproductive tract
(a) Ovum before fertilization (d) Rete testis
(b) Ovum after fertilization 52. Hormones secreted by the placenta to maintain
pregnancy are :
(c) Sperm after fertilization
(a) hCG, hPL, progestogens, prolactin
(d) Sperm before fertilization
(b) hCG, hPL, estrogens, relaxin, oxytocin
46. Which of the following cells during gametogenesis is (c) hCG, hPL, progestogens, estrogens
normally diploid ? :
(d) hCG, progestogens, estrogens, glycocorticoids
(a) Spermatid (b) Spermatogonia
53. The difference between spermiogenesis and
(c) Secondary polar body (d) Primary polar body spermiation is :
47. Hysterectomy is surgical removal of : (a) In spermiogenesis spermatids are formed, while in
(a) Prostate gland (b) Vas deferens spermiation spermatozoa are formed
(c) Mammary glands (d) Uterus (b) In spermiogenesis spermatozoa are formed, while
in spermiation spermatids are formed
48. Fertilization in humans is practically feasible only if :
(a) the sperms are transported into vagina just after (c) In spermiogenesis spermatozoa from Sertoli cells
the release of ovum in fallopian tube are released into the cavity of seminiferous
tubules, while in spermiation spermatozoa are
(b) the ovum and sperms are transported
formed
simultaneously to ampullary - isthmic junction of
the fallopian tube (d) In spermiogenesis spermatozoa are formed, while
in spermiation spermatozoa are released from
(c) the ovum and sperms are transported
Sertoli cells into the cavity of seminiferous tubules
simultaneously to ampullary - isthmic junction of
the cervix
830 NCERT Biology Booster

Answers (Section-E)

1. (b) 2. (d) 3. (d) 4. (a) 5. (d) 6. (d) 7. (d) 8. (c) 9. (b) 10. (d)
11. (d) 12. (c) 13. (c) 14. (b) 15. (c) 16. (b) 17. (d) 18. (c) 19. (a) 20. (c)
21. (c) 22. (c) 23. (a) 24. (c) 25. (a) 26. (c) 27. (b) 28. (a) 29. (b) 30. (b)
31. (d) 32. (b) 33. (c) 34. (c) 35. (b) 36. (a) 37. (a) 38. (b) 39. (a) 40. (b)
41. (a) 42. (b) 43. (c) 44. (a) 45. (d) 46. (b) 47. (d) 48. (b) 49. (c) 50. (b)
51. (c) 52. (c) 53. (d)
Human Reproduction 831

Hints

Section-A
2. (c) Testis is oval in shape, with a length of about 4-5 73. (d) Options (a), (b) and (c) are correct.
cm and a width of about 2-3 cm. Scrotum helps in 89. (b) Statement (iii) is incorrect.
maintaining the low temperature of testes, 92. (a) Corpus callosum is a thick band of nerve fibres
2-2.5°C, lower than the normal body temperature that divides the cerebral cortex lobes into left and
necessary for spermatogenesis. The spermato- right hemispheres.
gonia are present on the inside wall of 96. (c)
seminiferous tubules, farthest from the lumen.
5. (a) Statements (ii), (iii) and (iv) are correct.
Proliferative/follicular Luteal/secretory
7. (a) The region outside the seminiferous tubules called phase phase
interstitial spaces possess Leydig cells/interstitial (5th to 15th day) (15th to 29th day)
cells. These cells secrete androgens (testosterone).
8. (d) Options (a) and (b) are correct.
10. (d) Since, seminiferous tubules produce sperms, so it Menstruation
(1st to 5th day)
is not an accessory duct.
Menstrual Cycle
19. (a) Statements (i) and (v) are incorrect.
23. (d) Options (a), (b) and (c) are correct. 97. (b) Ovulation occurs during the middle of each
25. (c) Ovary produces steroid hormones like oestrogen menstrual cycle. Considering the options
and progesterone and protein hormones like provided, the middle of cycle falls on 17th day.
relaxin and inhibin. 98. (a) Since, ovulation occurs during the middle of each
Relaxin is secreted by corpus luteum only druing cycle, i. e., 14th day (considering the standard
the later stages of pregnancy to soften ligaments, menstrual cycle of 28/29 days).
especially those that hold the pubic symphysis 106. (d) A is LH which is a protein hormone which causes
together. ovulation.
Inhibin is also secreted by corpus luteum and it 114. (a) Implantation of embryo occurs in blastocyst stage.
inhibits the FSH and GnRH production. 127. (a) After implantation embryo gets nutrition from
26. (b) Relaxin is a protein hormone. placenta.
37. (b) Only one uterus is present. Perimetrium of uterus 129. (d) Options (a), (b) and (c) are correct.
wall is external and thin. 130. (d) Options (a), (b) and (c) are incorrect.
38. (d) Options (a), (b) and (c) are correct. 136. (c) hCG, hPL and relaxin.
51. (d) Options (a), (b) and (c) are correct. 137. (b) During pregnancy, the levels of thyroxin,
58. (a) Oogonia or female gamete mother cell are formed prolactin, cortisol, estrogen and progesterone
within each foetal ovary. No more oogonia are decrease. High levels of LH, FSH and hCG do not
formed and added after birth. stimulate endometrial thickening. High levels of
60. (b) Statements (i) and (v) are incorrect. LH and FSH induces ovulation.
68. (d) Options (a), (b) and (c) are correct. 138. (c) Prolactin is secreted by pituitary gland.
69. (d) Spermatogonium is diploid and spermatid is 146. (a) Oxytocin is injected to cause uterine contraction to
haploid. induce labor.
71. (b) Spermatogonia, primary spermatocyte, primary 148. (a) Oxytocin is released by posterior pituitary.
oocyte and oogonia have 46 chromosomes as
these are diploid structures.

Section-B
1. (c) Polyspermy in mammals is prevented due to of reception, it causes opening of Na+ cannels to
depolarisation of ovum membrane. As the sperm cause depolarisation of ovum membrane (fast
head comes in contact with fertilisation cone/cone block to check polyspermy) and Ca 2+ move into
832 NCERT Biology Booster

the egg. Sperm and ovum membrane dissolve. 14. (d) LH is secreted by anterior pituitary and not by
Complete sperm enters cytoplasm of egg and the placenta.
envelope is left out. Ca 2+ influx causes extrusion 16. (a) Zygote undergoes cleavage (mitotic divisions) to
of cortical granules (cortical reaction) and zona form 2, 4, 8, and 16 daughter cells called
reactions which makes zona pellucida blastomeres. It is called morula at 8-16 celled
impermeable to second sperm by destroying stage and it continues to divide to form blastocyst.
sperm receptors. 17. (a) The membranous covering of the ovum at
Cortical reaction and zona reaction constitute ovulation is called corona radiata. A radiately
slow block to check polyspermy. striated membrane situated next to the yolk of an
7. (a) As the head of sperm binds to zona pelucida of ovum is called zona radiata. The thick transparent
ovum, the acrosomal cap releases its content by membrane surrounding an ovum before
exocytosis which includes hyaluronidase and implantation is called zona pellucida. Chorion is
corona penetrating enzymes. This is termed the outermost foetal membrane around embryo.
acrosomal reaction. This reaction helps the sperm 18. (a) Fimbriae, infundibulum and isthumus along with
to reach the plasma membrane of ovum by ampula are parts of oviduct (fallopian tube). Labia
dissolving corona radiata and zona pellucida. majora are fleshy folds of tissue, which extends
8. (b) Ampulla is a part of oviduct (fallopian tube) down from the mons pubis and surround the
between infundibulum and isthmus. vaginal opening. Labia minora are paired folds of
9. (c) Spermatogonia increase in number by mitosis. tissue under the labia majora.
Primary spermatocyte undergo meiosis.
Spermatids get transformed into spermatozoa.

Section-C
1. (c) Human egg is alecithal, i. e., egg without yolk. 13. (c) Lack of menstruation may be indicative of
Centrolecithal eggs have yolk concentrated in the pregnancy. However, it may also be caused due to
centre of egg, e. g ., insects. some other underlying causes like stress, poor
3. (c) Corpus luteum secretes large amounts of health etc.
progesterone which is essential to maintain 18. (d) In menstrual cycle, follicular phase (proliferative
pregnancy. phase) is followed by luteal phase (secretory
10. (c) Expulsion of placenta is called ‘after birth’. phase).
Parturition is the process of delivery of foetus.

Section-D
1. (c) Chorion: It is the outermost membrane Yolk sac : It is present in the developing embryo.
surrounding an embryo. It contributes to the It is the source of blood cells, also act as shock
formation of placenta. absorber and prevents desiccation of embryo.
Allantois : It is the foeatal membrane lying 5. (a) Statements (ii) and (iv) are correct.
below chorion and forms a part of placenta. 9. (b) Blastocyst comes in contact with the endometrium
Amnion : It is the innermost membrane that in the region of embryonal knob/embryonal disc
encloses embryo and is filled with amniotic fluid. and adheres to it. The surface cells of trophoblast
It is not a part of placenta. Amniotic fluid serves as secrete lytic enzymes which cause corrosion of
shock absorber for foetus and prevents endometrial lining. They also give rise to
desiccation. finger-like outgrowths called villi. Villi helps in
fixation and absorption of nutrients.

You might also like